Myth of the Month: Does nitroglycerin response predict coronary artery disease?

Article Type
Changed
Fri, 01/18/2019 - 16:17

 

A 55-year-old man presents to the emergency department with substernal chest pain. The pain has occurred off and on over the past 2 hours. He has no family history of coronary artery disease. He has no history of diabetes, hypertension, or cigarette smoking. His most recent total cholesterol was 220 mg/dL (HDL, 40; LDL, 155). Blood pressure is 130/70. An ECG obtained on arrival is unremarkable. When he reached the ED, he received a nitroglycerin tablet with resolution of his pain within 4 minutes.

What is the most accurate statement?

A. The chance of CAD in this man over the next 10 years was 8% before his symptoms and is now greater than 20%.

B. The chance of CAD in this man over the next 10 years was 8% and is still 8%.

C. The chance of CAD in this man over the next 10 years was 15% before his symptoms and is now close to 100%.

D. The chance of CAD in this man over the next 10 years was 15% before his symptoms and is now close to 50%.

For years, giving nitroglycerin to patients who present with chest pain has been considered a good therapy, and the response to the medication has been considered a sign that the pain was likely due to cardiac ischemia. Is there evidence that this is true?

Dr. Douglas S. Paauw
Eric A. Shry, MD, and his colleagues looked at the usefulness of nitroglycerin response in the treatment of chest pain as a predictor of ischemic chest pain in an emergency department setting.1

The study was a retrospective review of 223 patients who presented to the ED over a 5-month period with ongoing chest pain. They looked at patients who had ongoing chest pain in the ED, received nitroglycerin, and did not receive any therapy other than aspirin within 10 minutes of receiving nitroglycerin. Nitroglycerin response was compared with the final diagnosis of cardiac versus noncardiac chest pain.

Of the patients with a final determination of cardiac chest pain, 88% had a nitroglycerin response, whereas 92% of the patients with noncardiac chest pain had a nitroglycerin response (P = .50).

Deborah B. Diercks, MD, and her colleagues looked at improvement in chest pain scores in the ED in patients treated with nitroglycerin and whether it correlated with a cardiac etiology of chest pain.2 The study was a prospective, observational study of 664 patients in an urban tertiary care ED over a 16-month period. An 11-point numeric chest pain scale was assessed and recorded by research assistants before and 5 minutes after receiving nitroglycerin. The scale ranged from 0 (no pain) to 10 (worst pain imaginable).

A final diagnosis of a cardiac etiology for chest pain was found in 18% of the patients in the study. Of the patients who had cardiac-related chest pain, 20% had no reduction in pain with nitroglycerin, compared with 19% of the patients without cardiac-related chest pain. Complete or significant reduction in chest pain occurred with nitroglycerin in 31% of patients with cardiac chest pain and 27% of the patients without cardiac chest pain (P = .76).

Two other studies with similar designs showed similar results. Robert Steele, MD, and his colleagues studied 270 patients in a prospective observational cohort study of patients with chest pain presenting to an urban ED.3 Patients presenting to the ED with active chest pain who received nitroglycerin were enrolled.

The sensitivity in this study for nitroglycerin relief determining cardiac chest pain was 72%, and the specificity was 37%, with a positive likelihood ratio for coronary artery disease if nitroglycerin response of 1.1 (0.96-1.34).

In another prospective, observational cohort study, 459 patients who presented to an ED with chest pain were evaluated for response to nitroglycerin as a marker for ischemic cardiac disease.4 In this study, presence of ischemic cardiac disease was defined as diagnosis in the ED or during a 4-month follow-up period. Nitroglycerin relieved chest pain in 35% of patients who had coronary disease, whereas 41% of patients without coronary disease had a nitroglycerin response. This study had a much lower overall nitroglycerin response rate than any of the other studies.

Katherine Grailey, MD, and Paul Glasziou, MD, PhD, published a meta-analysis of nitroglycerin use for the diagnosis of chest pain, using the above referenced studies. They concluded that in the acute setting, nitroglycerin is not a reliable test of treatment for use in diagnosis of coronary artery disease.5

High response rate for nitroglycerin in the noncoronary artery groups in the studies may be due to a strong placebo effect and/or that nitroglycerin may help with pain caused by esophageal spasm. The lack of specificity in the pain relief response for nitroglycerin makes it not a helpful test. Note that all the studies have been in the acute, ED setting for chest pain. In the case presented at the beginning of the article, the response the patient had to nitroglycerin would not change the probability that he has coronary artery disease.
 

 

 

References

1. Am J Cardiol. 2002 Dec 1;90(11):1264-6.

2. Ann Emerg Med. 2005 Jun;45(6):581-5.

3. CJEM. 2006 May;8(3):164-9.

4. Ann Intern Med. 2003 Dec 16;139(12):979-86.

5. Emerg Med J. 2012 Mar;29(3):173-6.

Dr. Paauw is professor of medicine in the division of general internal medicine at the University of Washington, Seattle, and he serves as third-year medical student clerkship director at the University of Washington. Contact Dr. Paauw at [email protected] .

Issue
Emergency Medicine - 12(6)
Publications
Topics
Sections

 

A 55-year-old man presents to the emergency department with substernal chest pain. The pain has occurred off and on over the past 2 hours. He has no family history of coronary artery disease. He has no history of diabetes, hypertension, or cigarette smoking. His most recent total cholesterol was 220 mg/dL (HDL, 40; LDL, 155). Blood pressure is 130/70. An ECG obtained on arrival is unremarkable. When he reached the ED, he received a nitroglycerin tablet with resolution of his pain within 4 minutes.

What is the most accurate statement?

A. The chance of CAD in this man over the next 10 years was 8% before his symptoms and is now greater than 20%.

B. The chance of CAD in this man over the next 10 years was 8% and is still 8%.

C. The chance of CAD in this man over the next 10 years was 15% before his symptoms and is now close to 100%.

D. The chance of CAD in this man over the next 10 years was 15% before his symptoms and is now close to 50%.

For years, giving nitroglycerin to patients who present with chest pain has been considered a good therapy, and the response to the medication has been considered a sign that the pain was likely due to cardiac ischemia. Is there evidence that this is true?

Dr. Douglas S. Paauw
Eric A. Shry, MD, and his colleagues looked at the usefulness of nitroglycerin response in the treatment of chest pain as a predictor of ischemic chest pain in an emergency department setting.1

The study was a retrospective review of 223 patients who presented to the ED over a 5-month period with ongoing chest pain. They looked at patients who had ongoing chest pain in the ED, received nitroglycerin, and did not receive any therapy other than aspirin within 10 minutes of receiving nitroglycerin. Nitroglycerin response was compared with the final diagnosis of cardiac versus noncardiac chest pain.

Of the patients with a final determination of cardiac chest pain, 88% had a nitroglycerin response, whereas 92% of the patients with noncardiac chest pain had a nitroglycerin response (P = .50).

Deborah B. Diercks, MD, and her colleagues looked at improvement in chest pain scores in the ED in patients treated with nitroglycerin and whether it correlated with a cardiac etiology of chest pain.2 The study was a prospective, observational study of 664 patients in an urban tertiary care ED over a 16-month period. An 11-point numeric chest pain scale was assessed and recorded by research assistants before and 5 minutes after receiving nitroglycerin. The scale ranged from 0 (no pain) to 10 (worst pain imaginable).

A final diagnosis of a cardiac etiology for chest pain was found in 18% of the patients in the study. Of the patients who had cardiac-related chest pain, 20% had no reduction in pain with nitroglycerin, compared with 19% of the patients without cardiac-related chest pain. Complete or significant reduction in chest pain occurred with nitroglycerin in 31% of patients with cardiac chest pain and 27% of the patients without cardiac chest pain (P = .76).

Two other studies with similar designs showed similar results. Robert Steele, MD, and his colleagues studied 270 patients in a prospective observational cohort study of patients with chest pain presenting to an urban ED.3 Patients presenting to the ED with active chest pain who received nitroglycerin were enrolled.

The sensitivity in this study for nitroglycerin relief determining cardiac chest pain was 72%, and the specificity was 37%, with a positive likelihood ratio for coronary artery disease if nitroglycerin response of 1.1 (0.96-1.34).

In another prospective, observational cohort study, 459 patients who presented to an ED with chest pain were evaluated for response to nitroglycerin as a marker for ischemic cardiac disease.4 In this study, presence of ischemic cardiac disease was defined as diagnosis in the ED or during a 4-month follow-up period. Nitroglycerin relieved chest pain in 35% of patients who had coronary disease, whereas 41% of patients without coronary disease had a nitroglycerin response. This study had a much lower overall nitroglycerin response rate than any of the other studies.

Katherine Grailey, MD, and Paul Glasziou, MD, PhD, published a meta-analysis of nitroglycerin use for the diagnosis of chest pain, using the above referenced studies. They concluded that in the acute setting, nitroglycerin is not a reliable test of treatment for use in diagnosis of coronary artery disease.5

High response rate for nitroglycerin in the noncoronary artery groups in the studies may be due to a strong placebo effect and/or that nitroglycerin may help with pain caused by esophageal spasm. The lack of specificity in the pain relief response for nitroglycerin makes it not a helpful test. Note that all the studies have been in the acute, ED setting for chest pain. In the case presented at the beginning of the article, the response the patient had to nitroglycerin would not change the probability that he has coronary artery disease.
 

 

 

References

1. Am J Cardiol. 2002 Dec 1;90(11):1264-6.

2. Ann Emerg Med. 2005 Jun;45(6):581-5.

3. CJEM. 2006 May;8(3):164-9.

4. Ann Intern Med. 2003 Dec 16;139(12):979-86.

5. Emerg Med J. 2012 Mar;29(3):173-6.

Dr. Paauw is professor of medicine in the division of general internal medicine at the University of Washington, Seattle, and he serves as third-year medical student clerkship director at the University of Washington. Contact Dr. Paauw at [email protected] .

 

A 55-year-old man presents to the emergency department with substernal chest pain. The pain has occurred off and on over the past 2 hours. He has no family history of coronary artery disease. He has no history of diabetes, hypertension, or cigarette smoking. His most recent total cholesterol was 220 mg/dL (HDL, 40; LDL, 155). Blood pressure is 130/70. An ECG obtained on arrival is unremarkable. When he reached the ED, he received a nitroglycerin tablet with resolution of his pain within 4 minutes.

What is the most accurate statement?

A. The chance of CAD in this man over the next 10 years was 8% before his symptoms and is now greater than 20%.

B. The chance of CAD in this man over the next 10 years was 8% and is still 8%.

C. The chance of CAD in this man over the next 10 years was 15% before his symptoms and is now close to 100%.

D. The chance of CAD in this man over the next 10 years was 15% before his symptoms and is now close to 50%.

For years, giving nitroglycerin to patients who present with chest pain has been considered a good therapy, and the response to the medication has been considered a sign that the pain was likely due to cardiac ischemia. Is there evidence that this is true?

Dr. Douglas S. Paauw
Eric A. Shry, MD, and his colleagues looked at the usefulness of nitroglycerin response in the treatment of chest pain as a predictor of ischemic chest pain in an emergency department setting.1

The study was a retrospective review of 223 patients who presented to the ED over a 5-month period with ongoing chest pain. They looked at patients who had ongoing chest pain in the ED, received nitroglycerin, and did not receive any therapy other than aspirin within 10 minutes of receiving nitroglycerin. Nitroglycerin response was compared with the final diagnosis of cardiac versus noncardiac chest pain.

Of the patients with a final determination of cardiac chest pain, 88% had a nitroglycerin response, whereas 92% of the patients with noncardiac chest pain had a nitroglycerin response (P = .50).

Deborah B. Diercks, MD, and her colleagues looked at improvement in chest pain scores in the ED in patients treated with nitroglycerin and whether it correlated with a cardiac etiology of chest pain.2 The study was a prospective, observational study of 664 patients in an urban tertiary care ED over a 16-month period. An 11-point numeric chest pain scale was assessed and recorded by research assistants before and 5 minutes after receiving nitroglycerin. The scale ranged from 0 (no pain) to 10 (worst pain imaginable).

A final diagnosis of a cardiac etiology for chest pain was found in 18% of the patients in the study. Of the patients who had cardiac-related chest pain, 20% had no reduction in pain with nitroglycerin, compared with 19% of the patients without cardiac-related chest pain. Complete or significant reduction in chest pain occurred with nitroglycerin in 31% of patients with cardiac chest pain and 27% of the patients without cardiac chest pain (P = .76).

Two other studies with similar designs showed similar results. Robert Steele, MD, and his colleagues studied 270 patients in a prospective observational cohort study of patients with chest pain presenting to an urban ED.3 Patients presenting to the ED with active chest pain who received nitroglycerin were enrolled.

The sensitivity in this study for nitroglycerin relief determining cardiac chest pain was 72%, and the specificity was 37%, with a positive likelihood ratio for coronary artery disease if nitroglycerin response of 1.1 (0.96-1.34).

In another prospective, observational cohort study, 459 patients who presented to an ED with chest pain were evaluated for response to nitroglycerin as a marker for ischemic cardiac disease.4 In this study, presence of ischemic cardiac disease was defined as diagnosis in the ED or during a 4-month follow-up period. Nitroglycerin relieved chest pain in 35% of patients who had coronary disease, whereas 41% of patients without coronary disease had a nitroglycerin response. This study had a much lower overall nitroglycerin response rate than any of the other studies.

Katherine Grailey, MD, and Paul Glasziou, MD, PhD, published a meta-analysis of nitroglycerin use for the diagnosis of chest pain, using the above referenced studies. They concluded that in the acute setting, nitroglycerin is not a reliable test of treatment for use in diagnosis of coronary artery disease.5

High response rate for nitroglycerin in the noncoronary artery groups in the studies may be due to a strong placebo effect and/or that nitroglycerin may help with pain caused by esophageal spasm. The lack of specificity in the pain relief response for nitroglycerin makes it not a helpful test. Note that all the studies have been in the acute, ED setting for chest pain. In the case presented at the beginning of the article, the response the patient had to nitroglycerin would not change the probability that he has coronary artery disease.
 

 

 

References

1. Am J Cardiol. 2002 Dec 1;90(11):1264-6.

2. Ann Emerg Med. 2005 Jun;45(6):581-5.

3. CJEM. 2006 May;8(3):164-9.

4. Ann Intern Med. 2003 Dec 16;139(12):979-86.

5. Emerg Med J. 2012 Mar;29(3):173-6.

Dr. Paauw is professor of medicine in the division of general internal medicine at the University of Washington, Seattle, and he serves as third-year medical student clerkship director at the University of Washington. Contact Dr. Paauw at [email protected] .

Issue
Emergency Medicine - 12(6)
Issue
Emergency Medicine - 12(6)
Publications
Publications
Topics
Article Type
Sections
Disallow All Ads

Addressing sex and gender inequality in biomedical research

Article Type
Changed
Thu, 03/28/2019 - 15:01


Despite some advances in sex/gender equality in the biomedical sciences, significant inequalities persist in two general areas. Not only are biologic sex and gender insufficiently reported within research studies, but women are also underrepresented as basic and clinical researchers in academic medicine. While these issues may seem unrelated, addressing both will diversify knowledge and interdisciplinary research teams, as well as improve the value of the science produced and ultimately the quality of health care provided.

In 1986, the National Institutes of Health instituted a policy urging the inclusion of women as subjects in clinical trials. This policy became law when Congress passed the NIH Revitalization Act of 1993, which requires that NIH-supported clinical research include women and minorities as subjects “in approximately equal numbers of both sexes … unless different proportions are appropriate because of known prevalence, incidence, morbidity, mortality rates, or expected intervention effect.” Women of childbearing potential cannot be routinely excluded without a strong scientific rationale.

Dr. Stacie Geller
Created in 1990, the Office of Research on Women’s Health (ORWH) has played a critical role to strengthen research related to diseases, disorders, and conditions that affect women and ensure that women are appropriately represented in biomedical and behavioral research supported by NIH. ORWH has provided critical funding for basic research and clinical trials to study women’s health and explore the role of sex and gender differences in health outcomes.

Despite these initiatives, evidence suggests that sex/gender is still not sufficiently considered as a biologic variable in federally funded research, and studies oftentimes fail to account for the cultural and societal influences of gender in health outcomes. Women comprise more than half of clinical trial participants, yet 75% of federally funded studies published in 2009 failed to report any outcomes by sex/gender. Recent events, such as the Food and Drug Administration’s updated Ambien dosage recommendation for men versus women, demonstrate the harmful effects of failing to account for sex as a biologic variable.

In recognition of the slow progress, the NIH required that research grants submitted after Jan. 25, 2016, address biologic sex within their research design and added reviewer criteria related to consideration of biologic sex in the research proposal. Ensuring enhanced inclusion, analysis, and reporting of sex and gender goes beyond NIH policy to include NIH enforcement of its own policies. In addition, journal editors should add review criteria related to sex and gender, and researchers themselves should examine potential sex/gender differences in their research.

Abby Koch
The knowledge produced in biomedical research depends on the questions we ask and the answers we get depend on who is asking the questions. In order to increase analysis and reporting in research by sex and gender, it is necessary to expand the proportion of women leading research programs. The latest data from the Association of American Medical Colleges (2013-2014) shows that, although 47% of medical school students are female, only 21% of full professors, 15% of department chairs, and 16% of U.S. medical school deans are women. Gender diversity within academic research teams produces higher quality science, according to the results of some studies. Diversity also fosters innovation, contributes to greater creativity, balances biases, and brings new perspective to problems.

The NIH and ORWH have implemented various programs to diversify the sciences; however, change has been less than desired. Studies indicate that females have lower publication rates throughout their careers, and are less likely to receive an R01 than men, despite reporting equal likelihood of applying for R01 awards. Additionally, the intraorganizational and network reach of female scientists is smaller than that of men, hindering opportunities for collaboration and publication. Even in the instance of equally qualified men and women conducting comparable work, investigators find differential pay between male and female researchers, as well as differential promotion to leadership positions. These factors, both in part caused by and exacerbated by unconscious or implicit interpersonal and institutional biases, lead to higher female attrition within the sciences and academia.

Addressing disparities and promoting greater inclusion includes unmasking unconscious bias and putting greater efforts toward mentoring and leadership initiatives for women. Only by partnering efforts to increase inclusion of sex/gender within research design with efforts to diversify the biomedical workforce can we adequately consider the role of sex and gender in biomedical research.
 

Dr. Geller is the G. William Arends Professor of Obstetrics and Gynecology at the University of Illinois College of Medicine, and the Director of the UIC Center for Research on Women and Gender. Ms. Koch is a senior research specialist at the Center for Research on Women and Gender. They reported having no financial disclosures.

Publications
Topics
Sections


Despite some advances in sex/gender equality in the biomedical sciences, significant inequalities persist in two general areas. Not only are biologic sex and gender insufficiently reported within research studies, but women are also underrepresented as basic and clinical researchers in academic medicine. While these issues may seem unrelated, addressing both will diversify knowledge and interdisciplinary research teams, as well as improve the value of the science produced and ultimately the quality of health care provided.

In 1986, the National Institutes of Health instituted a policy urging the inclusion of women as subjects in clinical trials. This policy became law when Congress passed the NIH Revitalization Act of 1993, which requires that NIH-supported clinical research include women and minorities as subjects “in approximately equal numbers of both sexes … unless different proportions are appropriate because of known prevalence, incidence, morbidity, mortality rates, or expected intervention effect.” Women of childbearing potential cannot be routinely excluded without a strong scientific rationale.

Dr. Stacie Geller
Created in 1990, the Office of Research on Women’s Health (ORWH) has played a critical role to strengthen research related to diseases, disorders, and conditions that affect women and ensure that women are appropriately represented in biomedical and behavioral research supported by NIH. ORWH has provided critical funding for basic research and clinical trials to study women’s health and explore the role of sex and gender differences in health outcomes.

Despite these initiatives, evidence suggests that sex/gender is still not sufficiently considered as a biologic variable in federally funded research, and studies oftentimes fail to account for the cultural and societal influences of gender in health outcomes. Women comprise more than half of clinical trial participants, yet 75% of federally funded studies published in 2009 failed to report any outcomes by sex/gender. Recent events, such as the Food and Drug Administration’s updated Ambien dosage recommendation for men versus women, demonstrate the harmful effects of failing to account for sex as a biologic variable.

In recognition of the slow progress, the NIH required that research grants submitted after Jan. 25, 2016, address biologic sex within their research design and added reviewer criteria related to consideration of biologic sex in the research proposal. Ensuring enhanced inclusion, analysis, and reporting of sex and gender goes beyond NIH policy to include NIH enforcement of its own policies. In addition, journal editors should add review criteria related to sex and gender, and researchers themselves should examine potential sex/gender differences in their research.

Abby Koch
The knowledge produced in biomedical research depends on the questions we ask and the answers we get depend on who is asking the questions. In order to increase analysis and reporting in research by sex and gender, it is necessary to expand the proportion of women leading research programs. The latest data from the Association of American Medical Colleges (2013-2014) shows that, although 47% of medical school students are female, only 21% of full professors, 15% of department chairs, and 16% of U.S. medical school deans are women. Gender diversity within academic research teams produces higher quality science, according to the results of some studies. Diversity also fosters innovation, contributes to greater creativity, balances biases, and brings new perspective to problems.

The NIH and ORWH have implemented various programs to diversify the sciences; however, change has been less than desired. Studies indicate that females have lower publication rates throughout their careers, and are less likely to receive an R01 than men, despite reporting equal likelihood of applying for R01 awards. Additionally, the intraorganizational and network reach of female scientists is smaller than that of men, hindering opportunities for collaboration and publication. Even in the instance of equally qualified men and women conducting comparable work, investigators find differential pay between male and female researchers, as well as differential promotion to leadership positions. These factors, both in part caused by and exacerbated by unconscious or implicit interpersonal and institutional biases, lead to higher female attrition within the sciences and academia.

Addressing disparities and promoting greater inclusion includes unmasking unconscious bias and putting greater efforts toward mentoring and leadership initiatives for women. Only by partnering efforts to increase inclusion of sex/gender within research design with efforts to diversify the biomedical workforce can we adequately consider the role of sex and gender in biomedical research.
 

Dr. Geller is the G. William Arends Professor of Obstetrics and Gynecology at the University of Illinois College of Medicine, and the Director of the UIC Center for Research on Women and Gender. Ms. Koch is a senior research specialist at the Center for Research on Women and Gender. They reported having no financial disclosures.


Despite some advances in sex/gender equality in the biomedical sciences, significant inequalities persist in two general areas. Not only are biologic sex and gender insufficiently reported within research studies, but women are also underrepresented as basic and clinical researchers in academic medicine. While these issues may seem unrelated, addressing both will diversify knowledge and interdisciplinary research teams, as well as improve the value of the science produced and ultimately the quality of health care provided.

In 1986, the National Institutes of Health instituted a policy urging the inclusion of women as subjects in clinical trials. This policy became law when Congress passed the NIH Revitalization Act of 1993, which requires that NIH-supported clinical research include women and minorities as subjects “in approximately equal numbers of both sexes … unless different proportions are appropriate because of known prevalence, incidence, morbidity, mortality rates, or expected intervention effect.” Women of childbearing potential cannot be routinely excluded without a strong scientific rationale.

Dr. Stacie Geller
Created in 1990, the Office of Research on Women’s Health (ORWH) has played a critical role to strengthen research related to diseases, disorders, and conditions that affect women and ensure that women are appropriately represented in biomedical and behavioral research supported by NIH. ORWH has provided critical funding for basic research and clinical trials to study women’s health and explore the role of sex and gender differences in health outcomes.

Despite these initiatives, evidence suggests that sex/gender is still not sufficiently considered as a biologic variable in federally funded research, and studies oftentimes fail to account for the cultural and societal influences of gender in health outcomes. Women comprise more than half of clinical trial participants, yet 75% of federally funded studies published in 2009 failed to report any outcomes by sex/gender. Recent events, such as the Food and Drug Administration’s updated Ambien dosage recommendation for men versus women, demonstrate the harmful effects of failing to account for sex as a biologic variable.

In recognition of the slow progress, the NIH required that research grants submitted after Jan. 25, 2016, address biologic sex within their research design and added reviewer criteria related to consideration of biologic sex in the research proposal. Ensuring enhanced inclusion, analysis, and reporting of sex and gender goes beyond NIH policy to include NIH enforcement of its own policies. In addition, journal editors should add review criteria related to sex and gender, and researchers themselves should examine potential sex/gender differences in their research.

Abby Koch
The knowledge produced in biomedical research depends on the questions we ask and the answers we get depend on who is asking the questions. In order to increase analysis and reporting in research by sex and gender, it is necessary to expand the proportion of women leading research programs. The latest data from the Association of American Medical Colleges (2013-2014) shows that, although 47% of medical school students are female, only 21% of full professors, 15% of department chairs, and 16% of U.S. medical school deans are women. Gender diversity within academic research teams produces higher quality science, according to the results of some studies. Diversity also fosters innovation, contributes to greater creativity, balances biases, and brings new perspective to problems.

The NIH and ORWH have implemented various programs to diversify the sciences; however, change has been less than desired. Studies indicate that females have lower publication rates throughout their careers, and are less likely to receive an R01 than men, despite reporting equal likelihood of applying for R01 awards. Additionally, the intraorganizational and network reach of female scientists is smaller than that of men, hindering opportunities for collaboration and publication. Even in the instance of equally qualified men and women conducting comparable work, investigators find differential pay between male and female researchers, as well as differential promotion to leadership positions. These factors, both in part caused by and exacerbated by unconscious or implicit interpersonal and institutional biases, lead to higher female attrition within the sciences and academia.

Addressing disparities and promoting greater inclusion includes unmasking unconscious bias and putting greater efforts toward mentoring and leadership initiatives for women. Only by partnering efforts to increase inclusion of sex/gender within research design with efforts to diversify the biomedical workforce can we adequately consider the role of sex and gender in biomedical research.
 

Dr. Geller is the G. William Arends Professor of Obstetrics and Gynecology at the University of Illinois College of Medicine, and the Director of the UIC Center for Research on Women and Gender. Ms. Koch is a senior research specialist at the Center for Research on Women and Gender. They reported having no financial disclosures.

Publications
Publications
Topics
Article Type
Sections
Disallow All Ads

Change in end-of-life cancer care imperative

Article Type
Changed
Fri, 01/18/2019 - 16:16

 

With the passage of the Medicare Access and CHIP Reauthorization Act, changes to how cancer care is delivered are fast approaching. This legislation aims to reward value-based care and incentivize alternative payment models that prize quality. The shift from quantity-based to value-based reimbursement is motivated in part by the rising cost of health care as well as the growing demand from patients, employers, and payers to better understand the quality of care being delivered. In cancer care, one area of high-cost and questionable value being examined is aggressive care at the end of life.

Dr. Bobby Daly
Research has found that high-intensity end-of-life care, including intensive care unit use, improves neither survival nor quality of life for advanced cancer patients. There is also considerable variation in end-of-life care and this variation signals that there is opportunity for improvement. In a study examining site of death for patients with cancer in seven developed countries, Bekelman et al. found that 27% of decedents in the United States were admitted to the ICU in the last 30 days of life, more than twice the rate of other countries. The National Quality Forum endorses ICU admissions in the last 30 days of life as a marker of poor-quality cancer care. We examined oncology patient deaths in the ICU, and our results were published recently in the Journal of Oncology Practice. Though a small sample size, our multidisciplinary review found that nearly half of these ICU deaths were potentially avoidable with different medical management. A significant number identified as clinically avoidable were due to absent or insufficient advance care planning. In this patient population, only 25% had documented advance directives and only 13% had an outpatient palliative care evaluation.

Dr. Andrew Hantel
Innovative models in cancer care delivery are taking steps to address some of these deficits and improve care delivery for cancer patients. The Centers for Medicare and Medicaid Services recently launched the Oncology Care Model. The model seeks to improve care coordination with one goal being fewer avoidable hospitalizations and better end-of-life care. Participating Oncology Care Model practices must formulate a care plan that contains advance care planning documentation. Despite hospitals and professional societies, such as the American Society of Clinical Oncology, highlighting the importance of advance care planning, a recent study found no growth in the past decade in key advance care planning domains, such as discussion of end-of-life care preferences. Prior experiments in cancer care delivery, such as The Oncology Medical Home, have shown that patient-centered innovations, such as extended clinic hours, weekend services, and symptom algorithms can also result in health care savings by keeping patients out of the emergency room, hospital, and ICU at the end of life.

Dr. Blase Polite
In addition to innovation in care delivery, technological advances also have the potential to improve care for advanced cancer patients. A recent randomized trial showed an improvement in survival and quality of life for patients with stage III/IV lung cancer assigned to a mobile friendly web application that allowed them to self-assess and electronically report their symptoms to their oncologist, compared with standard care follow-up. Based on the reported symptoms, the computer algorithm was able to trigger early supportive care as needed. The study author reported, “This approach introduces a new era of follow-up in which patients can give and receive continuous feedback between visits to their oncologist.” Other innovations, including decision support based on big data sets, integration of evidence-based clinical pathways into the electronic health record, and improved tools for prognosis and timing of palliative care referrals also hold promise to improve care delivery for advanced cancer patients.

The scientific pace of progress in cancer care is exciting, with 19 therapies approved or granted a new indication in 2015. New categories of drugs, such as immunotherapies, are changing how we treat patients. It is also a time of great change in how cancer care is being delivered in our clinics, hospitals, and academic institutions. We must be vigilant in learning from these experiments in care delivery to ensure that they deliver on their promise of value to patients.

Dr. Bobby Daly, Dr. Andrew Hantel, and Dr. Blase Polite are with the University of Chicago.

Publications
Topics
Sections

 

With the passage of the Medicare Access and CHIP Reauthorization Act, changes to how cancer care is delivered are fast approaching. This legislation aims to reward value-based care and incentivize alternative payment models that prize quality. The shift from quantity-based to value-based reimbursement is motivated in part by the rising cost of health care as well as the growing demand from patients, employers, and payers to better understand the quality of care being delivered. In cancer care, one area of high-cost and questionable value being examined is aggressive care at the end of life.

Dr. Bobby Daly
Research has found that high-intensity end-of-life care, including intensive care unit use, improves neither survival nor quality of life for advanced cancer patients. There is also considerable variation in end-of-life care and this variation signals that there is opportunity for improvement. In a study examining site of death for patients with cancer in seven developed countries, Bekelman et al. found that 27% of decedents in the United States were admitted to the ICU in the last 30 days of life, more than twice the rate of other countries. The National Quality Forum endorses ICU admissions in the last 30 days of life as a marker of poor-quality cancer care. We examined oncology patient deaths in the ICU, and our results were published recently in the Journal of Oncology Practice. Though a small sample size, our multidisciplinary review found that nearly half of these ICU deaths were potentially avoidable with different medical management. A significant number identified as clinically avoidable were due to absent or insufficient advance care planning. In this patient population, only 25% had documented advance directives and only 13% had an outpatient palliative care evaluation.

Dr. Andrew Hantel
Innovative models in cancer care delivery are taking steps to address some of these deficits and improve care delivery for cancer patients. The Centers for Medicare and Medicaid Services recently launched the Oncology Care Model. The model seeks to improve care coordination with one goal being fewer avoidable hospitalizations and better end-of-life care. Participating Oncology Care Model practices must formulate a care plan that contains advance care planning documentation. Despite hospitals and professional societies, such as the American Society of Clinical Oncology, highlighting the importance of advance care planning, a recent study found no growth in the past decade in key advance care planning domains, such as discussion of end-of-life care preferences. Prior experiments in cancer care delivery, such as The Oncology Medical Home, have shown that patient-centered innovations, such as extended clinic hours, weekend services, and symptom algorithms can also result in health care savings by keeping patients out of the emergency room, hospital, and ICU at the end of life.

Dr. Blase Polite
In addition to innovation in care delivery, technological advances also have the potential to improve care for advanced cancer patients. A recent randomized trial showed an improvement in survival and quality of life for patients with stage III/IV lung cancer assigned to a mobile friendly web application that allowed them to self-assess and electronically report their symptoms to their oncologist, compared with standard care follow-up. Based on the reported symptoms, the computer algorithm was able to trigger early supportive care as needed. The study author reported, “This approach introduces a new era of follow-up in which patients can give and receive continuous feedback between visits to their oncologist.” Other innovations, including decision support based on big data sets, integration of evidence-based clinical pathways into the electronic health record, and improved tools for prognosis and timing of palliative care referrals also hold promise to improve care delivery for advanced cancer patients.

The scientific pace of progress in cancer care is exciting, with 19 therapies approved or granted a new indication in 2015. New categories of drugs, such as immunotherapies, are changing how we treat patients. It is also a time of great change in how cancer care is being delivered in our clinics, hospitals, and academic institutions. We must be vigilant in learning from these experiments in care delivery to ensure that they deliver on their promise of value to patients.

Dr. Bobby Daly, Dr. Andrew Hantel, and Dr. Blase Polite are with the University of Chicago.

 

With the passage of the Medicare Access and CHIP Reauthorization Act, changes to how cancer care is delivered are fast approaching. This legislation aims to reward value-based care and incentivize alternative payment models that prize quality. The shift from quantity-based to value-based reimbursement is motivated in part by the rising cost of health care as well as the growing demand from patients, employers, and payers to better understand the quality of care being delivered. In cancer care, one area of high-cost and questionable value being examined is aggressive care at the end of life.

Dr. Bobby Daly
Research has found that high-intensity end-of-life care, including intensive care unit use, improves neither survival nor quality of life for advanced cancer patients. There is also considerable variation in end-of-life care and this variation signals that there is opportunity for improvement. In a study examining site of death for patients with cancer in seven developed countries, Bekelman et al. found that 27% of decedents in the United States were admitted to the ICU in the last 30 days of life, more than twice the rate of other countries. The National Quality Forum endorses ICU admissions in the last 30 days of life as a marker of poor-quality cancer care. We examined oncology patient deaths in the ICU, and our results were published recently in the Journal of Oncology Practice. Though a small sample size, our multidisciplinary review found that nearly half of these ICU deaths were potentially avoidable with different medical management. A significant number identified as clinically avoidable were due to absent or insufficient advance care planning. In this patient population, only 25% had documented advance directives and only 13% had an outpatient palliative care evaluation.

Dr. Andrew Hantel
Innovative models in cancer care delivery are taking steps to address some of these deficits and improve care delivery for cancer patients. The Centers for Medicare and Medicaid Services recently launched the Oncology Care Model. The model seeks to improve care coordination with one goal being fewer avoidable hospitalizations and better end-of-life care. Participating Oncology Care Model practices must formulate a care plan that contains advance care planning documentation. Despite hospitals and professional societies, such as the American Society of Clinical Oncology, highlighting the importance of advance care planning, a recent study found no growth in the past decade in key advance care planning domains, such as discussion of end-of-life care preferences. Prior experiments in cancer care delivery, such as The Oncology Medical Home, have shown that patient-centered innovations, such as extended clinic hours, weekend services, and symptom algorithms can also result in health care savings by keeping patients out of the emergency room, hospital, and ICU at the end of life.

Dr. Blase Polite
In addition to innovation in care delivery, technological advances also have the potential to improve care for advanced cancer patients. A recent randomized trial showed an improvement in survival and quality of life for patients with stage III/IV lung cancer assigned to a mobile friendly web application that allowed them to self-assess and electronically report their symptoms to their oncologist, compared with standard care follow-up. Based on the reported symptoms, the computer algorithm was able to trigger early supportive care as needed. The study author reported, “This approach introduces a new era of follow-up in which patients can give and receive continuous feedback between visits to their oncologist.” Other innovations, including decision support based on big data sets, integration of evidence-based clinical pathways into the electronic health record, and improved tools for prognosis and timing of palliative care referrals also hold promise to improve care delivery for advanced cancer patients.

The scientific pace of progress in cancer care is exciting, with 19 therapies approved or granted a new indication in 2015. New categories of drugs, such as immunotherapies, are changing how we treat patients. It is also a time of great change in how cancer care is being delivered in our clinics, hospitals, and academic institutions. We must be vigilant in learning from these experiments in care delivery to ensure that they deliver on their promise of value to patients.

Dr. Bobby Daly, Dr. Andrew Hantel, and Dr. Blase Polite are with the University of Chicago.

Publications
Publications
Topics
Article Type
Sections
Disallow All Ads

Clown therapy is community psychiatry in disguise

Article Type
Changed
Mon, 04/16/2018 - 13:57


Ever had a patient who not only got better but used the insights she gained from talking with you to help others in distress? I have just such a patient in the Peruvian Amazon.

I’ve previously written about an annual clown trip to Peru that I make with my friend Patch Adams, MD, and 100 other humanitarian clowns from all over the world. We have been going there for a decade to spread cheer, and revitalize the impoverished community of Belén, which is situated in the Amazon floodplain in the city of Iquitos. We conduct workshops, perform street theater, create community art installations, visit hospitals, and work with local grass-roots organizations. For the last 5 years, we also have been conducting mental health clinics in the streets.

To provide a brief overview ... we go to a neighborhood, set up our space, and walk the streets with a bull horn. We announce our presence – “we are mental health professionals, and we’re meeting over at” ... and we talk with anybody, young and old, who wants to discuss health problems, family issues, or other concerns.

We sit in a public place and speak to individuals/couples/families for 20 minutes, while around us, support clowns entertain the kids. We neither make diagnoses nor give drugs; we come with a clown nose and an open heart, and we listen actively without judgment and focus on solutions. We help people identify their strengths and resilience, and give them practical advice. This is community psychiatry disguised as “clown therapy,” which is just another phrase for solution-oriented therapy/ positive psychology/reality therapy/resilience-based therapy, logotherapy, existential psychotherapy, or kitchen table wisdom. These street clinics have had a profound impact on patients and clinicians.

Three years ago, I met a middle-aged woman who was suicidally depressed, and together we negotiated a successful intervention. In summary, she emerged from a church that happened to be across the street from where we were setting up our clinic. A clown saw her weeping and approached her, and after talking with her assured her that there was somebody here right now – a mental health professional – who would talk with her.

Maria sat down and told an unbelievably painful story that was happening within her family. On that day, after 8 months of prayer and receiving no sign from God, she had decided to kill herself. After listening to her, I actually believed she could do it.

There are no treatment centers or emergency shelters for the poor in Belén, so at our closure, I made her promise that she would not try to kill herself until I could see her again at our next clinic 2 days away, and close by. I gave her an amulet that was blessed and told her it was a reminder of her promise, and that my smiling face would be with her until she saw me again. She returned with her daughters to the next clinic, and together, they found a way to take a step forward.

Last year, I made my first home visit, and met with Maria, her daughters, and new grandson in their “new” home where they were happily sustaining themselves . When I left, the love and appreciation was so overwhelming that I told them as long as I returned I would come visit every year.

I just got back from this year’s annual visit, and was again greeted with passionate tears of joy. We sat and talked, and Maria told me her story. It seems that people in the community were now coming to her as a resource when they were deeply depressed. People know that she had walked a similar path and moved beyond it.

Courtesy Dr. Hammerschlag
Dr. Carl A. Hammerschlag receives assistance in announcing that the clowns are back in Belen, Peru, for their annual mental health clinics.


She is a warm, good listener, and tells them a story about walking out of church and deciding she wanted to kill herself, and meeting with a tall gringo, a clown/doctor who miraculously saved her life. She gives them simple, practical advice, tells them how important it is to stay connected to their children, speak your truth with them openly; to pray for miracles and recognize them when they occur. She tells them to reach out for help, and people will reach out to them. She is a credible, inspiring friend who gives hope.

For those who remember when community psychiatry was actually a subspecialty, this is my vision of community mental health: People talking to credible witnesses/healers/resources in their community, whom they respect, who will listen without judgment, and maybe even say something that inspires a light in the darkness. It’s at least as effective as psychotropic drugs, and all its benefits come without side effects.

Once a year we come together, listen to each other’s stories, and continue our healing work together. Maria tells me her friends want to meet me. “They want to steal you away,” she says, “but I tell them I am not afraid.”

Maria, a lay therapist of sorts, is the community mental health consultant. Once a year, she consults with her gringo, the clown/doctor, to compare notes. We laugh and love, hug and cry, and give each other hope. No matter how divisive and polarizing the times, it is possible to come together in community and promote healing.

 

 

Dr. Hammerschlag is chief of community mental health of the Gesundheit! Institute and a faculty member at the University of Arizona, Phoenix. He is the author of several books on healing and spirituality, including “Kindling Spirit: Healing from Within” (New York: Turtle Island Press, 2010) and “The Dancing Healers: A Doctor’s Journey of Healing With Native Americans” (San Francisco: Harper, 1988). Dr. Hammerschlag’s website is healingdoc.com.

Publications
Topics
Sections


Ever had a patient who not only got better but used the insights she gained from talking with you to help others in distress? I have just such a patient in the Peruvian Amazon.

I’ve previously written about an annual clown trip to Peru that I make with my friend Patch Adams, MD, and 100 other humanitarian clowns from all over the world. We have been going there for a decade to spread cheer, and revitalize the impoverished community of Belén, which is situated in the Amazon floodplain in the city of Iquitos. We conduct workshops, perform street theater, create community art installations, visit hospitals, and work with local grass-roots organizations. For the last 5 years, we also have been conducting mental health clinics in the streets.

To provide a brief overview ... we go to a neighborhood, set up our space, and walk the streets with a bull horn. We announce our presence – “we are mental health professionals, and we’re meeting over at” ... and we talk with anybody, young and old, who wants to discuss health problems, family issues, or other concerns.

We sit in a public place and speak to individuals/couples/families for 20 minutes, while around us, support clowns entertain the kids. We neither make diagnoses nor give drugs; we come with a clown nose and an open heart, and we listen actively without judgment and focus on solutions. We help people identify their strengths and resilience, and give them practical advice. This is community psychiatry disguised as “clown therapy,” which is just another phrase for solution-oriented therapy/ positive psychology/reality therapy/resilience-based therapy, logotherapy, existential psychotherapy, or kitchen table wisdom. These street clinics have had a profound impact on patients and clinicians.

Three years ago, I met a middle-aged woman who was suicidally depressed, and together we negotiated a successful intervention. In summary, she emerged from a church that happened to be across the street from where we were setting up our clinic. A clown saw her weeping and approached her, and after talking with her assured her that there was somebody here right now – a mental health professional – who would talk with her.

Maria sat down and told an unbelievably painful story that was happening within her family. On that day, after 8 months of prayer and receiving no sign from God, she had decided to kill herself. After listening to her, I actually believed she could do it.

There are no treatment centers or emergency shelters for the poor in Belén, so at our closure, I made her promise that she would not try to kill herself until I could see her again at our next clinic 2 days away, and close by. I gave her an amulet that was blessed and told her it was a reminder of her promise, and that my smiling face would be with her until she saw me again. She returned with her daughters to the next clinic, and together, they found a way to take a step forward.

Last year, I made my first home visit, and met with Maria, her daughters, and new grandson in their “new” home where they were happily sustaining themselves . When I left, the love and appreciation was so overwhelming that I told them as long as I returned I would come visit every year.

I just got back from this year’s annual visit, and was again greeted with passionate tears of joy. We sat and talked, and Maria told me her story. It seems that people in the community were now coming to her as a resource when they were deeply depressed. People know that she had walked a similar path and moved beyond it.

Courtesy Dr. Hammerschlag
Dr. Carl A. Hammerschlag receives assistance in announcing that the clowns are back in Belen, Peru, for their annual mental health clinics.


She is a warm, good listener, and tells them a story about walking out of church and deciding she wanted to kill herself, and meeting with a tall gringo, a clown/doctor who miraculously saved her life. She gives them simple, practical advice, tells them how important it is to stay connected to their children, speak your truth with them openly; to pray for miracles and recognize them when they occur. She tells them to reach out for help, and people will reach out to them. She is a credible, inspiring friend who gives hope.

For those who remember when community psychiatry was actually a subspecialty, this is my vision of community mental health: People talking to credible witnesses/healers/resources in their community, whom they respect, who will listen without judgment, and maybe even say something that inspires a light in the darkness. It’s at least as effective as psychotropic drugs, and all its benefits come without side effects.

Once a year we come together, listen to each other’s stories, and continue our healing work together. Maria tells me her friends want to meet me. “They want to steal you away,” she says, “but I tell them I am not afraid.”

Maria, a lay therapist of sorts, is the community mental health consultant. Once a year, she consults with her gringo, the clown/doctor, to compare notes. We laugh and love, hug and cry, and give each other hope. No matter how divisive and polarizing the times, it is possible to come together in community and promote healing.

 

 

Dr. Hammerschlag is chief of community mental health of the Gesundheit! Institute and a faculty member at the University of Arizona, Phoenix. He is the author of several books on healing and spirituality, including “Kindling Spirit: Healing from Within” (New York: Turtle Island Press, 2010) and “The Dancing Healers: A Doctor’s Journey of Healing With Native Americans” (San Francisco: Harper, 1988). Dr. Hammerschlag’s website is healingdoc.com.


Ever had a patient who not only got better but used the insights she gained from talking with you to help others in distress? I have just such a patient in the Peruvian Amazon.

I’ve previously written about an annual clown trip to Peru that I make with my friend Patch Adams, MD, and 100 other humanitarian clowns from all over the world. We have been going there for a decade to spread cheer, and revitalize the impoverished community of Belén, which is situated in the Amazon floodplain in the city of Iquitos. We conduct workshops, perform street theater, create community art installations, visit hospitals, and work with local grass-roots organizations. For the last 5 years, we also have been conducting mental health clinics in the streets.

To provide a brief overview ... we go to a neighborhood, set up our space, and walk the streets with a bull horn. We announce our presence – “we are mental health professionals, and we’re meeting over at” ... and we talk with anybody, young and old, who wants to discuss health problems, family issues, or other concerns.

We sit in a public place and speak to individuals/couples/families for 20 minutes, while around us, support clowns entertain the kids. We neither make diagnoses nor give drugs; we come with a clown nose and an open heart, and we listen actively without judgment and focus on solutions. We help people identify their strengths and resilience, and give them practical advice. This is community psychiatry disguised as “clown therapy,” which is just another phrase for solution-oriented therapy/ positive psychology/reality therapy/resilience-based therapy, logotherapy, existential psychotherapy, or kitchen table wisdom. These street clinics have had a profound impact on patients and clinicians.

Three years ago, I met a middle-aged woman who was suicidally depressed, and together we negotiated a successful intervention. In summary, she emerged from a church that happened to be across the street from where we were setting up our clinic. A clown saw her weeping and approached her, and after talking with her assured her that there was somebody here right now – a mental health professional – who would talk with her.

Maria sat down and told an unbelievably painful story that was happening within her family. On that day, after 8 months of prayer and receiving no sign from God, she had decided to kill herself. After listening to her, I actually believed she could do it.

There are no treatment centers or emergency shelters for the poor in Belén, so at our closure, I made her promise that she would not try to kill herself until I could see her again at our next clinic 2 days away, and close by. I gave her an amulet that was blessed and told her it was a reminder of her promise, and that my smiling face would be with her until she saw me again. She returned with her daughters to the next clinic, and together, they found a way to take a step forward.

Last year, I made my first home visit, and met with Maria, her daughters, and new grandson in their “new” home where they were happily sustaining themselves . When I left, the love and appreciation was so overwhelming that I told them as long as I returned I would come visit every year.

I just got back from this year’s annual visit, and was again greeted with passionate tears of joy. We sat and talked, and Maria told me her story. It seems that people in the community were now coming to her as a resource when they were deeply depressed. People know that she had walked a similar path and moved beyond it.

Courtesy Dr. Hammerschlag
Dr. Carl A. Hammerschlag receives assistance in announcing that the clowns are back in Belen, Peru, for their annual mental health clinics.


She is a warm, good listener, and tells them a story about walking out of church and deciding she wanted to kill herself, and meeting with a tall gringo, a clown/doctor who miraculously saved her life. She gives them simple, practical advice, tells them how important it is to stay connected to their children, speak your truth with them openly; to pray for miracles and recognize them when they occur. She tells them to reach out for help, and people will reach out to them. She is a credible, inspiring friend who gives hope.

For those who remember when community psychiatry was actually a subspecialty, this is my vision of community mental health: People talking to credible witnesses/healers/resources in their community, whom they respect, who will listen without judgment, and maybe even say something that inspires a light in the darkness. It’s at least as effective as psychotropic drugs, and all its benefits come without side effects.

Once a year we come together, listen to each other’s stories, and continue our healing work together. Maria tells me her friends want to meet me. “They want to steal you away,” she says, “but I tell them I am not afraid.”

Maria, a lay therapist of sorts, is the community mental health consultant. Once a year, she consults with her gringo, the clown/doctor, to compare notes. We laugh and love, hug and cry, and give each other hope. No matter how divisive and polarizing the times, it is possible to come together in community and promote healing.

 

 

Dr. Hammerschlag is chief of community mental health of the Gesundheit! Institute and a faculty member at the University of Arizona, Phoenix. He is the author of several books on healing and spirituality, including “Kindling Spirit: Healing from Within” (New York: Turtle Island Press, 2010) and “The Dancing Healers: A Doctor’s Journey of Healing With Native Americans” (San Francisco: Harper, 1988). Dr. Hammerschlag’s website is healingdoc.com.

Publications
Publications
Topics
Article Type
Sections
Disallow All Ads

When is it time to stop hormone therapy?

Article Type
Changed
Fri, 01/18/2019 - 16:16

 

Extended use of systemic hormone therapy represents one area that clinicians commonly encounter. However, because randomized trial data are not available, helping women make decisions regarding long-term use of hormone therapy is controversial. When it is finalized, the 2016 hormone therapy position statement from the North American Menopause Society will address this topic.

Here I’m referring to the patient who likely started systemic hormone therapy when she was a younger, or more recently menopausal, woman (perhaps in her early 50s), but now she’s in her 60s. In my practice, a patient in this age range would likely be on a lower-than-standard dose of hormone therapy than what she began with originally.

And now the question is, Should she continue, or should she stop, hormone therapy? The median duration of bothersome symptoms is about 10 or 11 years, from the best available data – far longer than what many physicians assume.

Dr. Andrew M. Kaunitz
If a woman started hormone therapy in her 50s for bothersome menopausal symptoms and now she’s in her 60s, there’s a good chance that she may have outgrown her vasomotor symptoms. So what I do in my practice every year, or every other year, is encourage the patient to reduce the dose, and if bothersome symptoms do not return, then I continue the patient on that lower dose. Using this long-term dose-tapering strategy, when the patient is down to a very low dose (for instance a 0.025-mg to 0.0375-mg estradiol patch, 0.5 mg oral estradiol, or conjugated equine estrogen 0.3 mg) and symptoms have not returned, I discuss with her either stopping hormone therapy or continuing it, not for symptom relief, but for osteoporosis prevention. This is where the shared decision making comes in, because we don’t have high-quality randomized trial data to inform us.

And risk stratification also becomes relevant. Let’s say the patient is a slender white or Asian woman with a low body mass index , or she has a parent who had a hip fracture. Continuing low-dose systemic hormone therapy in this case, particularly for osteoporosis prevention when vasomotor symptoms are no longer present, might make sense. However, if the patient is obese, and, therefore has a lower risk for osteoporosis, it may be that ongoing use of systemic hormone therapy would not be indicated, and that patient should be encouraged to discontinue at that point.

Also, if a uterus is not present – and we’re talking only about estrogen therapy, given its greater safety profile with long-term use with respect to breast cancer – clinicians and women can be more comfortable with continued use of low-dose systemic hormone therapy for osteoporosis prevention. If a uterus is present, then women making decisions about long-term use of hormone therapy need to be aware of the small, but I believe real, elevated risk of breast cancer. With each office visit and when decisions about refilling prescriptions or continuing hormone therapy are made, this is an important issue to discuss, particularly if there’s an intact uterus. These discussions also need to be documented in the record.

What about the route of estrogen? Age, BMI, and oral estrogen therapy each represent independent risk factors for venous thromboembolism. For older, as well as obese menopausal women, who are candidates for systemic hormone therapy, I prefer transdermal over oral estrogen therapy.

Finally, I counsel women that although vasomotor symptoms/hot flashes improve as women age, the same is not true for genitourinary syndrome of menopause (GSM, also known as genital atrophy). If vaginal dryness, pain with sex, or other manifestations of GSM occur in women tapering their dose of systemic hormone therapy or in women who have discontinued systemic hormone therapy, initiation and long-term use of low-dose vaginal estrogen should be considered.

References

1. Menopause. 2014 Jun;21(6):679-81.

Dr. Kaunitz is a professor and associate chair of the department of obstetrics and gynecology, University of Florida in Jacksonville. He is on the board of trustees of the North American Menopause Society. He reports being a consultant or on the advisory board or review panel of several pharmaceutical companies, and receiving grant support from several pharmaceutical companies. He receives royalties from UpToDate.

Meeting/Event
Publications
Topics
Sections
Meeting/Event
Meeting/Event

 

Extended use of systemic hormone therapy represents one area that clinicians commonly encounter. However, because randomized trial data are not available, helping women make decisions regarding long-term use of hormone therapy is controversial. When it is finalized, the 2016 hormone therapy position statement from the North American Menopause Society will address this topic.

Here I’m referring to the patient who likely started systemic hormone therapy when she was a younger, or more recently menopausal, woman (perhaps in her early 50s), but now she’s in her 60s. In my practice, a patient in this age range would likely be on a lower-than-standard dose of hormone therapy than what she began with originally.

And now the question is, Should she continue, or should she stop, hormone therapy? The median duration of bothersome symptoms is about 10 or 11 years, from the best available data – far longer than what many physicians assume.

Dr. Andrew M. Kaunitz
If a woman started hormone therapy in her 50s for bothersome menopausal symptoms and now she’s in her 60s, there’s a good chance that she may have outgrown her vasomotor symptoms. So what I do in my practice every year, or every other year, is encourage the patient to reduce the dose, and if bothersome symptoms do not return, then I continue the patient on that lower dose. Using this long-term dose-tapering strategy, when the patient is down to a very low dose (for instance a 0.025-mg to 0.0375-mg estradiol patch, 0.5 mg oral estradiol, or conjugated equine estrogen 0.3 mg) and symptoms have not returned, I discuss with her either stopping hormone therapy or continuing it, not for symptom relief, but for osteoporosis prevention. This is where the shared decision making comes in, because we don’t have high-quality randomized trial data to inform us.

And risk stratification also becomes relevant. Let’s say the patient is a slender white or Asian woman with a low body mass index , or she has a parent who had a hip fracture. Continuing low-dose systemic hormone therapy in this case, particularly for osteoporosis prevention when vasomotor symptoms are no longer present, might make sense. However, if the patient is obese, and, therefore has a lower risk for osteoporosis, it may be that ongoing use of systemic hormone therapy would not be indicated, and that patient should be encouraged to discontinue at that point.

Also, if a uterus is not present – and we’re talking only about estrogen therapy, given its greater safety profile with long-term use with respect to breast cancer – clinicians and women can be more comfortable with continued use of low-dose systemic hormone therapy for osteoporosis prevention. If a uterus is present, then women making decisions about long-term use of hormone therapy need to be aware of the small, but I believe real, elevated risk of breast cancer. With each office visit and when decisions about refilling prescriptions or continuing hormone therapy are made, this is an important issue to discuss, particularly if there’s an intact uterus. These discussions also need to be documented in the record.

What about the route of estrogen? Age, BMI, and oral estrogen therapy each represent independent risk factors for venous thromboembolism. For older, as well as obese menopausal women, who are candidates for systemic hormone therapy, I prefer transdermal over oral estrogen therapy.

Finally, I counsel women that although vasomotor symptoms/hot flashes improve as women age, the same is not true for genitourinary syndrome of menopause (GSM, also known as genital atrophy). If vaginal dryness, pain with sex, or other manifestations of GSM occur in women tapering their dose of systemic hormone therapy or in women who have discontinued systemic hormone therapy, initiation and long-term use of low-dose vaginal estrogen should be considered.

References

1. Menopause. 2014 Jun;21(6):679-81.

Dr. Kaunitz is a professor and associate chair of the department of obstetrics and gynecology, University of Florida in Jacksonville. He is on the board of trustees of the North American Menopause Society. He reports being a consultant or on the advisory board or review panel of several pharmaceutical companies, and receiving grant support from several pharmaceutical companies. He receives royalties from UpToDate.

 

Extended use of systemic hormone therapy represents one area that clinicians commonly encounter. However, because randomized trial data are not available, helping women make decisions regarding long-term use of hormone therapy is controversial. When it is finalized, the 2016 hormone therapy position statement from the North American Menopause Society will address this topic.

Here I’m referring to the patient who likely started systemic hormone therapy when she was a younger, or more recently menopausal, woman (perhaps in her early 50s), but now she’s in her 60s. In my practice, a patient in this age range would likely be on a lower-than-standard dose of hormone therapy than what she began with originally.

And now the question is, Should she continue, or should she stop, hormone therapy? The median duration of bothersome symptoms is about 10 or 11 years, from the best available data – far longer than what many physicians assume.

Dr. Andrew M. Kaunitz
If a woman started hormone therapy in her 50s for bothersome menopausal symptoms and now she’s in her 60s, there’s a good chance that she may have outgrown her vasomotor symptoms. So what I do in my practice every year, or every other year, is encourage the patient to reduce the dose, and if bothersome symptoms do not return, then I continue the patient on that lower dose. Using this long-term dose-tapering strategy, when the patient is down to a very low dose (for instance a 0.025-mg to 0.0375-mg estradiol patch, 0.5 mg oral estradiol, or conjugated equine estrogen 0.3 mg) and symptoms have not returned, I discuss with her either stopping hormone therapy or continuing it, not for symptom relief, but for osteoporosis prevention. This is where the shared decision making comes in, because we don’t have high-quality randomized trial data to inform us.

And risk stratification also becomes relevant. Let’s say the patient is a slender white or Asian woman with a low body mass index , or she has a parent who had a hip fracture. Continuing low-dose systemic hormone therapy in this case, particularly for osteoporosis prevention when vasomotor symptoms are no longer present, might make sense. However, if the patient is obese, and, therefore has a lower risk for osteoporosis, it may be that ongoing use of systemic hormone therapy would not be indicated, and that patient should be encouraged to discontinue at that point.

Also, if a uterus is not present – and we’re talking only about estrogen therapy, given its greater safety profile with long-term use with respect to breast cancer – clinicians and women can be more comfortable with continued use of low-dose systemic hormone therapy for osteoporosis prevention. If a uterus is present, then women making decisions about long-term use of hormone therapy need to be aware of the small, but I believe real, elevated risk of breast cancer. With each office visit and when decisions about refilling prescriptions or continuing hormone therapy are made, this is an important issue to discuss, particularly if there’s an intact uterus. These discussions also need to be documented in the record.

What about the route of estrogen? Age, BMI, and oral estrogen therapy each represent independent risk factors for venous thromboembolism. For older, as well as obese menopausal women, who are candidates for systemic hormone therapy, I prefer transdermal over oral estrogen therapy.

Finally, I counsel women that although vasomotor symptoms/hot flashes improve as women age, the same is not true for genitourinary syndrome of menopause (GSM, also known as genital atrophy). If vaginal dryness, pain with sex, or other manifestations of GSM occur in women tapering their dose of systemic hormone therapy or in women who have discontinued systemic hormone therapy, initiation and long-term use of low-dose vaginal estrogen should be considered.

References

1. Menopause. 2014 Jun;21(6):679-81.

Dr. Kaunitz is a professor and associate chair of the department of obstetrics and gynecology, University of Florida in Jacksonville. He is on the board of trustees of the North American Menopause Society. He reports being a consultant or on the advisory board or review panel of several pharmaceutical companies, and receiving grant support from several pharmaceutical companies. He receives royalties from UpToDate.

Publications
Publications
Topics
Article Type
Sections
Disallow All Ads

Facing a medical board investigation

Article Type
Changed
Thu, 03/28/2019 - 15:01

 

(This column is the third of a three-part series.)

Question: The state medical board is requesting the medical records of an angry patient who has filed a formal complaint alleging incompetent care. Your immediate impulse is to rush off a letter defending your treatment – and to include a sarcastic remark about ingratitude.

However, you decided not to respond, choosing to await the scheduled interview with the board investigator. To your surprise, he turned out to be warm and friendly, and his relative youth reminded you of your son. Letting your guard down, you spoke freely during the meeting and showed him the medical records. Later, you appeared at the formal board hearing without legal counsel, as the complainant had not suffered any injury while under your care. Under this scenario, which of the following statements is best?

Dr. S.Y. Tan

A. You must never respond in writing to any board inquiry.

B. You should have called your insurance carrier immediately.

C. You trusted the investigator because what you told him could not be introduced as evidence.

D. You correctly assumed that, without injuries, there is no liability.

E. You did not insist on having an attorney, because this is not part of your due process rights.

Answer: B. Who gets into trouble with medical disciplinary boards? In a California study,1 the authors concluded that male physicians were nearly three times as likely to as women physicians, and those who were non–board certified twice as likely to as their board-certified counterparts. Obstetricians, gynecologists, family physicians, general practitioners, and psychiatrists were more likely than internists, whereas pediatricians and radiologists were the least likely. Age had a smaller influence (elevated risk with increasing age), and foreign medical graduates also had an increased risk.

It is true that many board complaints are dismissed and never investigated, but the number of state licensure actions in 2013 was almost four times greater than the number of malpractice payouts. Moreover, board actions against a doctor, unlike a malpractice lawsuit, are premised on substandard or unethical conduct, and do not require a showing of actual patient harm.

A recent news article bemoans the travails of a medical board encounter.2 According to a 2009 report, one of every eight physicians in California was being reported to the board each year, with a quarter of complaints leading to an investigation. In turn, a quarter of investigated complaints led to disciplinary proceedings against the physician.

A medical board investigation begins with a complaint lodged by a patient or some other party such as a pharmacist, nurse, or hospital peer review committee. Some states – e.g., California, Georgia, and Maryland – allow complaints to be anonymous, and many states now allow online submissions, whose ease may be expected to increase the number of filings. Oklahoma, for example, saw its board complaints rise by 40% in the 2 years after permitting this option.

Some complaints such as rudeness and disputes over fees may seem like a minor nuisance, but they can mushroom into new and more serious charges. The medical records may reveal other potential misconduct, such as delinquent record keeping or failure to obtain informed consent.

It is therefore prudent to treat all complaints seriously. A written response is necessary, but it must be cowritten with your attorney. Immediately contact your malpractice insurance carrier once an investigator approaches you, or upon notification by the medical board. Most malpractice policies include coverage for medical board investigations.

The simple rule is not to discuss with anyone – including (especially) the complainant – and await a call from the attorney assigned to you. Better yet, ask for an attorney whose skills you are aware of. Make sure he/she is experienced with board proceedings and not just malpractice litigation. There are legal technicalities that may be of importance, such as statutes of limitations barring board actions or the timely filing of an appeal. Everything should flow from here. The lawyer’s advice will most likely be: “Don’t speak to anyone, including your colleagues; don’t contact the patient or family; and don’t release any records without first consulting me.”

You should together draft a response letter. Whereas the attorney will be attentive to the legal ramifications, only you as the doctor can provide the clinical knowledge, context, and empathy. In formulating any written response where patient care is at issue, heed the following principles:

1. Be honest. Truth always surfaces in due course. Candor and trustworthiness are virtues expected of all doctors.

2. Be accurate. Review carefully all relevant medical records. Pay compulsive attention to factual accuracy.

3. Be focused. Do not ramble on and on regarding unrelated or tangential issues. Focus on what the complaint is about. No one is interested in your views regarding your philosophy of medical practice or the health care system – they do not belong in a complaint response letter.

4. Be humble. Arrogance at this stage may prove disastrous. Adopt a contrite and humble tone. Blame no one, especially the patient.

5. Be a patient advocate. Show that patient well-being is always your first and last concern.

 

 

In the preliminary stages, a board investigator may call the doctor, typically taking a friendly and casual approach, and this may lull the physician into saying more than is necessary or releasing the medical records. The shared information can come back to haunt the doctor when it is disclosed in a subsequent hearing.

In some situations, the board may offer an informal settlement conference to resolve the issue and obtain a “consent agreement” from the doctor. Depending on the facts – and the proposed settlement terms – your attorney may instead advise proceeding to a formal hearing to present exculpatory evidence and to confront the complainant.

Physicians are guaranteed the right of due process during an investigation. Timely notice, the right to a hearing, to confront the evidence, and to have legal representation are the basic due process rights. And boards must treat the physician fairly and reasonably.

However, where public harm is an imminent risk, boards have the power to immediately institute a temporary suspension of the doctor’s right to practice. Examples warranting such summary suspensions may include sexual misconduct, inappropriate opioid prescriptions, egregious negligent conduct, or impairment from alcohol or drug abuse.

Boards face a dilemma over these cases, because they need to balance depriving a doctor of due process rights, albeit temporarily, against allowing a bad doctor to pose a clear and present threat to the public. In Texas, 32 summary suspensions took place in 2011, a figure that fell to 13 in 2014.

The aggrieved physician typically has the right to appeal an adverse board decision to the courts, although “courts have no inherent appellate jurisdiction over official acts of administrative agencies except where the legislature has made some statutory provision for judicial review.”3

For example, under California’s Code of Civil Procedure section 1094.5, one has the right to have a Superior Court judge review the board’s decision to determine if there has been an abuse of discretion.

Despite the fact that board administrative proceedings are quasi criminal in nature, most states require only a “preponderance of evidence” to find the doctor guilty. This is the evidentiary standard used in civil cases where only monetary damages are at stake, yet it is used in determining whether a doctor would lose the liberty to practice his or her profession.

However, some jurisdictions – such as California, Florida, and Illinois – call for a higher threshold of guilt, requiring proof with “clear and convincing evidence.”

Finally, in the United Kingdom and countries such as Singapore, a guilty decision requires even more evidence – i.e., proof “beyond reasonable doubt” – which is the threshold required in criminal prosecutions.

References

1. Arch Intern Med. 2004 Mar 22;164(6):653-8.

2. Leigh Page. The Black Cloud of a Medical Board Investigation. Medscape, Dec 23, 2015.

3. Crane v. Cont’l Tel. Co. of Cal., 775 P.2d 705 (Nev. 1989).

Dr. Tan is emeritus professor of medicine and former adjunct professor of law at the University of Hawaii, and currently directs the St. Francis International Center for Healthcare Ethics in Honolulu. This article is meant to be educational and does not constitute medical, ethical, or legal advice. Some of the articles in this series are adapted from the author’s 2006 book, “Medical Malpractice: Understanding the Law, Managing the Risk,” and his 2012 Halsbury treatise, “Medical Negligence and Professional Misconduct.” For additional information, readers may contact the author at [email protected].

Publications
Topics
Sections

 

(This column is the third of a three-part series.)

Question: The state medical board is requesting the medical records of an angry patient who has filed a formal complaint alleging incompetent care. Your immediate impulse is to rush off a letter defending your treatment – and to include a sarcastic remark about ingratitude.

However, you decided not to respond, choosing to await the scheduled interview with the board investigator. To your surprise, he turned out to be warm and friendly, and his relative youth reminded you of your son. Letting your guard down, you spoke freely during the meeting and showed him the medical records. Later, you appeared at the formal board hearing without legal counsel, as the complainant had not suffered any injury while under your care. Under this scenario, which of the following statements is best?

Dr. S.Y. Tan

A. You must never respond in writing to any board inquiry.

B. You should have called your insurance carrier immediately.

C. You trusted the investigator because what you told him could not be introduced as evidence.

D. You correctly assumed that, without injuries, there is no liability.

E. You did not insist on having an attorney, because this is not part of your due process rights.

Answer: B. Who gets into trouble with medical disciplinary boards? In a California study,1 the authors concluded that male physicians were nearly three times as likely to as women physicians, and those who were non–board certified twice as likely to as their board-certified counterparts. Obstetricians, gynecologists, family physicians, general practitioners, and psychiatrists were more likely than internists, whereas pediatricians and radiologists were the least likely. Age had a smaller influence (elevated risk with increasing age), and foreign medical graduates also had an increased risk.

It is true that many board complaints are dismissed and never investigated, but the number of state licensure actions in 2013 was almost four times greater than the number of malpractice payouts. Moreover, board actions against a doctor, unlike a malpractice lawsuit, are premised on substandard or unethical conduct, and do not require a showing of actual patient harm.

A recent news article bemoans the travails of a medical board encounter.2 According to a 2009 report, one of every eight physicians in California was being reported to the board each year, with a quarter of complaints leading to an investigation. In turn, a quarter of investigated complaints led to disciplinary proceedings against the physician.

A medical board investigation begins with a complaint lodged by a patient or some other party such as a pharmacist, nurse, or hospital peer review committee. Some states – e.g., California, Georgia, and Maryland – allow complaints to be anonymous, and many states now allow online submissions, whose ease may be expected to increase the number of filings. Oklahoma, for example, saw its board complaints rise by 40% in the 2 years after permitting this option.

Some complaints such as rudeness and disputes over fees may seem like a minor nuisance, but they can mushroom into new and more serious charges. The medical records may reveal other potential misconduct, such as delinquent record keeping or failure to obtain informed consent.

It is therefore prudent to treat all complaints seriously. A written response is necessary, but it must be cowritten with your attorney. Immediately contact your malpractice insurance carrier once an investigator approaches you, or upon notification by the medical board. Most malpractice policies include coverage for medical board investigations.

The simple rule is not to discuss with anyone – including (especially) the complainant – and await a call from the attorney assigned to you. Better yet, ask for an attorney whose skills you are aware of. Make sure he/she is experienced with board proceedings and not just malpractice litigation. There are legal technicalities that may be of importance, such as statutes of limitations barring board actions or the timely filing of an appeal. Everything should flow from here. The lawyer’s advice will most likely be: “Don’t speak to anyone, including your colleagues; don’t contact the patient or family; and don’t release any records without first consulting me.”

You should together draft a response letter. Whereas the attorney will be attentive to the legal ramifications, only you as the doctor can provide the clinical knowledge, context, and empathy. In formulating any written response where patient care is at issue, heed the following principles:

1. Be honest. Truth always surfaces in due course. Candor and trustworthiness are virtues expected of all doctors.

2. Be accurate. Review carefully all relevant medical records. Pay compulsive attention to factual accuracy.

3. Be focused. Do not ramble on and on regarding unrelated or tangential issues. Focus on what the complaint is about. No one is interested in your views regarding your philosophy of medical practice or the health care system – they do not belong in a complaint response letter.

4. Be humble. Arrogance at this stage may prove disastrous. Adopt a contrite and humble tone. Blame no one, especially the patient.

5. Be a patient advocate. Show that patient well-being is always your first and last concern.

 

 

In the preliminary stages, a board investigator may call the doctor, typically taking a friendly and casual approach, and this may lull the physician into saying more than is necessary or releasing the medical records. The shared information can come back to haunt the doctor when it is disclosed in a subsequent hearing.

In some situations, the board may offer an informal settlement conference to resolve the issue and obtain a “consent agreement” from the doctor. Depending on the facts – and the proposed settlement terms – your attorney may instead advise proceeding to a formal hearing to present exculpatory evidence and to confront the complainant.

Physicians are guaranteed the right of due process during an investigation. Timely notice, the right to a hearing, to confront the evidence, and to have legal representation are the basic due process rights. And boards must treat the physician fairly and reasonably.

However, where public harm is an imminent risk, boards have the power to immediately institute a temporary suspension of the doctor’s right to practice. Examples warranting such summary suspensions may include sexual misconduct, inappropriate opioid prescriptions, egregious negligent conduct, or impairment from alcohol or drug abuse.

Boards face a dilemma over these cases, because they need to balance depriving a doctor of due process rights, albeit temporarily, against allowing a bad doctor to pose a clear and present threat to the public. In Texas, 32 summary suspensions took place in 2011, a figure that fell to 13 in 2014.

The aggrieved physician typically has the right to appeal an adverse board decision to the courts, although “courts have no inherent appellate jurisdiction over official acts of administrative agencies except where the legislature has made some statutory provision for judicial review.”3

For example, under California’s Code of Civil Procedure section 1094.5, one has the right to have a Superior Court judge review the board’s decision to determine if there has been an abuse of discretion.

Despite the fact that board administrative proceedings are quasi criminal in nature, most states require only a “preponderance of evidence” to find the doctor guilty. This is the evidentiary standard used in civil cases where only monetary damages are at stake, yet it is used in determining whether a doctor would lose the liberty to practice his or her profession.

However, some jurisdictions – such as California, Florida, and Illinois – call for a higher threshold of guilt, requiring proof with “clear and convincing evidence.”

Finally, in the United Kingdom and countries such as Singapore, a guilty decision requires even more evidence – i.e., proof “beyond reasonable doubt” – which is the threshold required in criminal prosecutions.

References

1. Arch Intern Med. 2004 Mar 22;164(6):653-8.

2. Leigh Page. The Black Cloud of a Medical Board Investigation. Medscape, Dec 23, 2015.

3. Crane v. Cont’l Tel. Co. of Cal., 775 P.2d 705 (Nev. 1989).

Dr. Tan is emeritus professor of medicine and former adjunct professor of law at the University of Hawaii, and currently directs the St. Francis International Center for Healthcare Ethics in Honolulu. This article is meant to be educational and does not constitute medical, ethical, or legal advice. Some of the articles in this series are adapted from the author’s 2006 book, “Medical Malpractice: Understanding the Law, Managing the Risk,” and his 2012 Halsbury treatise, “Medical Negligence and Professional Misconduct.” For additional information, readers may contact the author at [email protected].

 

(This column is the third of a three-part series.)

Question: The state medical board is requesting the medical records of an angry patient who has filed a formal complaint alleging incompetent care. Your immediate impulse is to rush off a letter defending your treatment – and to include a sarcastic remark about ingratitude.

However, you decided not to respond, choosing to await the scheduled interview with the board investigator. To your surprise, he turned out to be warm and friendly, and his relative youth reminded you of your son. Letting your guard down, you spoke freely during the meeting and showed him the medical records. Later, you appeared at the formal board hearing without legal counsel, as the complainant had not suffered any injury while under your care. Under this scenario, which of the following statements is best?

Dr. S.Y. Tan

A. You must never respond in writing to any board inquiry.

B. You should have called your insurance carrier immediately.

C. You trusted the investigator because what you told him could not be introduced as evidence.

D. You correctly assumed that, without injuries, there is no liability.

E. You did not insist on having an attorney, because this is not part of your due process rights.

Answer: B. Who gets into trouble with medical disciplinary boards? In a California study,1 the authors concluded that male physicians were nearly three times as likely to as women physicians, and those who were non–board certified twice as likely to as their board-certified counterparts. Obstetricians, gynecologists, family physicians, general practitioners, and psychiatrists were more likely than internists, whereas pediatricians and radiologists were the least likely. Age had a smaller influence (elevated risk with increasing age), and foreign medical graduates also had an increased risk.

It is true that many board complaints are dismissed and never investigated, but the number of state licensure actions in 2013 was almost four times greater than the number of malpractice payouts. Moreover, board actions against a doctor, unlike a malpractice lawsuit, are premised on substandard or unethical conduct, and do not require a showing of actual patient harm.

A recent news article bemoans the travails of a medical board encounter.2 According to a 2009 report, one of every eight physicians in California was being reported to the board each year, with a quarter of complaints leading to an investigation. In turn, a quarter of investigated complaints led to disciplinary proceedings against the physician.

A medical board investigation begins with a complaint lodged by a patient or some other party such as a pharmacist, nurse, or hospital peer review committee. Some states – e.g., California, Georgia, and Maryland – allow complaints to be anonymous, and many states now allow online submissions, whose ease may be expected to increase the number of filings. Oklahoma, for example, saw its board complaints rise by 40% in the 2 years after permitting this option.

Some complaints such as rudeness and disputes over fees may seem like a minor nuisance, but they can mushroom into new and more serious charges. The medical records may reveal other potential misconduct, such as delinquent record keeping or failure to obtain informed consent.

It is therefore prudent to treat all complaints seriously. A written response is necessary, but it must be cowritten with your attorney. Immediately contact your malpractice insurance carrier once an investigator approaches you, or upon notification by the medical board. Most malpractice policies include coverage for medical board investigations.

The simple rule is not to discuss with anyone – including (especially) the complainant – and await a call from the attorney assigned to you. Better yet, ask for an attorney whose skills you are aware of. Make sure he/she is experienced with board proceedings and not just malpractice litigation. There are legal technicalities that may be of importance, such as statutes of limitations barring board actions or the timely filing of an appeal. Everything should flow from here. The lawyer’s advice will most likely be: “Don’t speak to anyone, including your colleagues; don’t contact the patient or family; and don’t release any records without first consulting me.”

You should together draft a response letter. Whereas the attorney will be attentive to the legal ramifications, only you as the doctor can provide the clinical knowledge, context, and empathy. In formulating any written response where patient care is at issue, heed the following principles:

1. Be honest. Truth always surfaces in due course. Candor and trustworthiness are virtues expected of all doctors.

2. Be accurate. Review carefully all relevant medical records. Pay compulsive attention to factual accuracy.

3. Be focused. Do not ramble on and on regarding unrelated or tangential issues. Focus on what the complaint is about. No one is interested in your views regarding your philosophy of medical practice or the health care system – they do not belong in a complaint response letter.

4. Be humble. Arrogance at this stage may prove disastrous. Adopt a contrite and humble tone. Blame no one, especially the patient.

5. Be a patient advocate. Show that patient well-being is always your first and last concern.

 

 

In the preliminary stages, a board investigator may call the doctor, typically taking a friendly and casual approach, and this may lull the physician into saying more than is necessary or releasing the medical records. The shared information can come back to haunt the doctor when it is disclosed in a subsequent hearing.

In some situations, the board may offer an informal settlement conference to resolve the issue and obtain a “consent agreement” from the doctor. Depending on the facts – and the proposed settlement terms – your attorney may instead advise proceeding to a formal hearing to present exculpatory evidence and to confront the complainant.

Physicians are guaranteed the right of due process during an investigation. Timely notice, the right to a hearing, to confront the evidence, and to have legal representation are the basic due process rights. And boards must treat the physician fairly and reasonably.

However, where public harm is an imminent risk, boards have the power to immediately institute a temporary suspension of the doctor’s right to practice. Examples warranting such summary suspensions may include sexual misconduct, inappropriate opioid prescriptions, egregious negligent conduct, or impairment from alcohol or drug abuse.

Boards face a dilemma over these cases, because they need to balance depriving a doctor of due process rights, albeit temporarily, against allowing a bad doctor to pose a clear and present threat to the public. In Texas, 32 summary suspensions took place in 2011, a figure that fell to 13 in 2014.

The aggrieved physician typically has the right to appeal an adverse board decision to the courts, although “courts have no inherent appellate jurisdiction over official acts of administrative agencies except where the legislature has made some statutory provision for judicial review.”3

For example, under California’s Code of Civil Procedure section 1094.5, one has the right to have a Superior Court judge review the board’s decision to determine if there has been an abuse of discretion.

Despite the fact that board administrative proceedings are quasi criminal in nature, most states require only a “preponderance of evidence” to find the doctor guilty. This is the evidentiary standard used in civil cases where only monetary damages are at stake, yet it is used in determining whether a doctor would lose the liberty to practice his or her profession.

However, some jurisdictions – such as California, Florida, and Illinois – call for a higher threshold of guilt, requiring proof with “clear and convincing evidence.”

Finally, in the United Kingdom and countries such as Singapore, a guilty decision requires even more evidence – i.e., proof “beyond reasonable doubt” – which is the threshold required in criminal prosecutions.

References

1. Arch Intern Med. 2004 Mar 22;164(6):653-8.

2. Leigh Page. The Black Cloud of a Medical Board Investigation. Medscape, Dec 23, 2015.

3. Crane v. Cont’l Tel. Co. of Cal., 775 P.2d 705 (Nev. 1989).

Dr. Tan is emeritus professor of medicine and former adjunct professor of law at the University of Hawaii, and currently directs the St. Francis International Center for Healthcare Ethics in Honolulu. This article is meant to be educational and does not constitute medical, ethical, or legal advice. Some of the articles in this series are adapted from the author’s 2006 book, “Medical Malpractice: Understanding the Law, Managing the Risk,” and his 2012 Halsbury treatise, “Medical Negligence and Professional Misconduct.” For additional information, readers may contact the author at [email protected].

Publications
Publications
Topics
Article Type
Sections
Disallow All Ads

The Problem of ‘Is’ and ‘Ought’ for Surgeons

Article Type
Changed
Wed, 01/02/2019 - 09:40
Display Headline
The Problem of ‘Is’ and ‘Ought’ for Surgeons

Many years ago during medical school, I took time out to pursue graduate studies in philosophy. At that time, I took a number of courses that explored various approaches to the philosophical questions of morality and ethics. My ultimate goal, even back then, was to focus on ethical issues in the practice of medicine.

I often found the philosophical discussions from the “giants” in philosophy were not always easy to apply to everyday problems. After completing my graduate studies in philosophy and nearing the end of medical school, I found that I was drawn to surgery. Not surprisingly, many surgical faculty that I interviewed with for my residency saw little application of my philosophy studies to the practice of surgery. Although I felt confident that ethics was central to the practice of surgery, I let pass the general suggestions from many senior surgeons that surgery and philosophical analysis have little in common.

 

Dr. Peter Angelos

In recent years, however, I have increasingly seen an area of overlap that I believe will be central to the future of surgery. The options for the treatment of critically ill surgical patients across all areas of surgery have increased dramatically. Just in the area of cardiovascular disease, patients with failing hearts have the option of mechanical assist devices. Patients with multiple comorbidities and vascular problems can have numerous endovascular procedures done that years ago would have been unthinkable. Consider a patient with a ventricular assist device on a ventilator who is being dialyzed. Such a patient may be supported for weeks or months beyond what was possible just a few decades ago.

Whereas our surgical forefathers were constantly asking the question, “What can be done for this patient?” those caring for critically ill patients today must repeatedly ask, “What should we do for this patient?” Years ago, the statement, “there is nothing more that we can offer” was much more commonly heard than it is today. The critical question for today – “What should be done?” – is often more challenging and nuanced than “what can be done?” Whenever we ask “what should be done?” we must take into account the values of the patient and weigh the possible outcomes and the inherent risks of the possible interventions with the patient’s goals.

The current necessity to answer “what should be done?” has several striking parallels with the classical philosophical problem of “is” and “ought.” Over the centuries, many philosophers have considered whether we can derive an “ought” from an “is.” In other words, just because one can show that something is the case in the world, it does not automatically follow that it ought to be that way. David Hume, the Scottish philosopher (1711-1776), famously argued that there is a tremendous difference between statements about what is and statements about what ought to be. In particular, Hume argued that we cannot logically derive an “ought” from an “is.”

Despite the centuries that have passed since Hume’s days, I believe that his analysis has much to teach modern surgery. Just because we can undertake many interventions for our patients, it does not follow that we should undertake all of those interventions. A central aspect of what many of us refer to commonly as “surgical judgment” is deciding among the many possible interventions for a patient, what specific ones ought we offer. Although this entire discussion may seem theoretical (and possibly even arcane) to some surgeons, I firmly believe that one of the greatest challenges to the future of surgery is whether surgeons are willing to address the question of what should be done for every patient.

Excellent surgeons have traditionally been seen as having both technical mastery and sound judgment. In the current era in which surgeons are increasingly pushed to do more cases and maximize RVUs, multiple forces are encouraging surgeons to increasingly become pure technicians. Technicians can answer the question “what can be done?” However, “what should be done for this specific patient?” is a question that only a physician can answer. In the decades to come, we must ensure that surgeons continue to engage in the harder questions of “what should be done?” so that we do not forget that “is” and “ought” are different. The mastery of surgery involves not only the technical expertise that can be applied on behalf of a patient, but also the appreciation and understanding of the patient’s values so that surgeons can make recommendations about what should be done for their patients.

Dr. Angelos is the Linda Kohler Anderson Professor of Surgery and Surgical Ethics; chief, endocrine surgery; and associate director of the MacLean Center for Clinical Medical Ethics at the University of Chicago.

Publications
Sections

Many years ago during medical school, I took time out to pursue graduate studies in philosophy. At that time, I took a number of courses that explored various approaches to the philosophical questions of morality and ethics. My ultimate goal, even back then, was to focus on ethical issues in the practice of medicine.

I often found the philosophical discussions from the “giants” in philosophy were not always easy to apply to everyday problems. After completing my graduate studies in philosophy and nearing the end of medical school, I found that I was drawn to surgery. Not surprisingly, many surgical faculty that I interviewed with for my residency saw little application of my philosophy studies to the practice of surgery. Although I felt confident that ethics was central to the practice of surgery, I let pass the general suggestions from many senior surgeons that surgery and philosophical analysis have little in common.

 

Dr. Peter Angelos

In recent years, however, I have increasingly seen an area of overlap that I believe will be central to the future of surgery. The options for the treatment of critically ill surgical patients across all areas of surgery have increased dramatically. Just in the area of cardiovascular disease, patients with failing hearts have the option of mechanical assist devices. Patients with multiple comorbidities and vascular problems can have numerous endovascular procedures done that years ago would have been unthinkable. Consider a patient with a ventricular assist device on a ventilator who is being dialyzed. Such a patient may be supported for weeks or months beyond what was possible just a few decades ago.

Whereas our surgical forefathers were constantly asking the question, “What can be done for this patient?” those caring for critically ill patients today must repeatedly ask, “What should we do for this patient?” Years ago, the statement, “there is nothing more that we can offer” was much more commonly heard than it is today. The critical question for today – “What should be done?” – is often more challenging and nuanced than “what can be done?” Whenever we ask “what should be done?” we must take into account the values of the patient and weigh the possible outcomes and the inherent risks of the possible interventions with the patient’s goals.

The current necessity to answer “what should be done?” has several striking parallels with the classical philosophical problem of “is” and “ought.” Over the centuries, many philosophers have considered whether we can derive an “ought” from an “is.” In other words, just because one can show that something is the case in the world, it does not automatically follow that it ought to be that way. David Hume, the Scottish philosopher (1711-1776), famously argued that there is a tremendous difference between statements about what is and statements about what ought to be. In particular, Hume argued that we cannot logically derive an “ought” from an “is.”

Despite the centuries that have passed since Hume’s days, I believe that his analysis has much to teach modern surgery. Just because we can undertake many interventions for our patients, it does not follow that we should undertake all of those interventions. A central aspect of what many of us refer to commonly as “surgical judgment” is deciding among the many possible interventions for a patient, what specific ones ought we offer. Although this entire discussion may seem theoretical (and possibly even arcane) to some surgeons, I firmly believe that one of the greatest challenges to the future of surgery is whether surgeons are willing to address the question of what should be done for every patient.

Excellent surgeons have traditionally been seen as having both technical mastery and sound judgment. In the current era in which surgeons are increasingly pushed to do more cases and maximize RVUs, multiple forces are encouraging surgeons to increasingly become pure technicians. Technicians can answer the question “what can be done?” However, “what should be done for this specific patient?” is a question that only a physician can answer. In the decades to come, we must ensure that surgeons continue to engage in the harder questions of “what should be done?” so that we do not forget that “is” and “ought” are different. The mastery of surgery involves not only the technical expertise that can be applied on behalf of a patient, but also the appreciation and understanding of the patient’s values so that surgeons can make recommendations about what should be done for their patients.

Dr. Angelos is the Linda Kohler Anderson Professor of Surgery and Surgical Ethics; chief, endocrine surgery; and associate director of the MacLean Center for Clinical Medical Ethics at the University of Chicago.

Many years ago during medical school, I took time out to pursue graduate studies in philosophy. At that time, I took a number of courses that explored various approaches to the philosophical questions of morality and ethics. My ultimate goal, even back then, was to focus on ethical issues in the practice of medicine.

I often found the philosophical discussions from the “giants” in philosophy were not always easy to apply to everyday problems. After completing my graduate studies in philosophy and nearing the end of medical school, I found that I was drawn to surgery. Not surprisingly, many surgical faculty that I interviewed with for my residency saw little application of my philosophy studies to the practice of surgery. Although I felt confident that ethics was central to the practice of surgery, I let pass the general suggestions from many senior surgeons that surgery and philosophical analysis have little in common.

 

Dr. Peter Angelos

In recent years, however, I have increasingly seen an area of overlap that I believe will be central to the future of surgery. The options for the treatment of critically ill surgical patients across all areas of surgery have increased dramatically. Just in the area of cardiovascular disease, patients with failing hearts have the option of mechanical assist devices. Patients with multiple comorbidities and vascular problems can have numerous endovascular procedures done that years ago would have been unthinkable. Consider a patient with a ventricular assist device on a ventilator who is being dialyzed. Such a patient may be supported for weeks or months beyond what was possible just a few decades ago.

Whereas our surgical forefathers were constantly asking the question, “What can be done for this patient?” those caring for critically ill patients today must repeatedly ask, “What should we do for this patient?” Years ago, the statement, “there is nothing more that we can offer” was much more commonly heard than it is today. The critical question for today – “What should be done?” – is often more challenging and nuanced than “what can be done?” Whenever we ask “what should be done?” we must take into account the values of the patient and weigh the possible outcomes and the inherent risks of the possible interventions with the patient’s goals.

The current necessity to answer “what should be done?” has several striking parallels with the classical philosophical problem of “is” and “ought.” Over the centuries, many philosophers have considered whether we can derive an “ought” from an “is.” In other words, just because one can show that something is the case in the world, it does not automatically follow that it ought to be that way. David Hume, the Scottish philosopher (1711-1776), famously argued that there is a tremendous difference between statements about what is and statements about what ought to be. In particular, Hume argued that we cannot logically derive an “ought” from an “is.”

Despite the centuries that have passed since Hume’s days, I believe that his analysis has much to teach modern surgery. Just because we can undertake many interventions for our patients, it does not follow that we should undertake all of those interventions. A central aspect of what many of us refer to commonly as “surgical judgment” is deciding among the many possible interventions for a patient, what specific ones ought we offer. Although this entire discussion may seem theoretical (and possibly even arcane) to some surgeons, I firmly believe that one of the greatest challenges to the future of surgery is whether surgeons are willing to address the question of what should be done for every patient.

Excellent surgeons have traditionally been seen as having both technical mastery and sound judgment. In the current era in which surgeons are increasingly pushed to do more cases and maximize RVUs, multiple forces are encouraging surgeons to increasingly become pure technicians. Technicians can answer the question “what can be done?” However, “what should be done for this specific patient?” is a question that only a physician can answer. In the decades to come, we must ensure that surgeons continue to engage in the harder questions of “what should be done?” so that we do not forget that “is” and “ought” are different. The mastery of surgery involves not only the technical expertise that can be applied on behalf of a patient, but also the appreciation and understanding of the patient’s values so that surgeons can make recommendations about what should be done for their patients.

Dr. Angelos is the Linda Kohler Anderson Professor of Surgery and Surgical Ethics; chief, endocrine surgery; and associate director of the MacLean Center for Clinical Medical Ethics at the University of Chicago.

Publications
Publications
Article Type
Display Headline
The Problem of ‘Is’ and ‘Ought’ for Surgeons
Display Headline
The Problem of ‘Is’ and ‘Ought’ for Surgeons
Sections
Disallow All Ads

Sic transit gloria mundi

Article Type
Changed
Wed, 01/02/2019 - 09:40
Display Headline
Sic transit gloria mundi

The email came with the words, “It is with sadness we report that Frank Moody died. …” I was instantly transported to the last time I saw the man and a flood of emotions swept over me. The name Frank Moody will ring a distant bell or none at all to some in our profession. Like many of the greats of surgery, he belongs to the ages.

I remember the first time I asked a student, “Who is Michael DeBakey?” I was dumbfounded to be greeted with a blank stare. How could a student of medicine not know of Dr. DeBakey? A few years later, the same question prompted a smart aleck reply that he was the man who invented DeBakey forceps. Well, of course he did invent the forceps, but to know nothing further of the man who was the world’s expert on ulcer disease in the 1940s, the progenitor of the National Medical Library, and among the foremost pioneers of heart surgery seemed beyond belief.

 

Photo credit: Dwight Andrews/UTHealth
Dr. Frank Moody

My mentor, Ernest Poulos, has long since left the active surgical scene. At times he would note the passing of one of his heroes like Carl Moyer (look it up!) and say, “Sic transit gloria mundi.” At 27 and anxious to get the right to cut into my fellow human beings, I would cock my head like a confounded puppy and wonder what that meant. I looked up the translation and meaning long ago, but now with age I understand the phrase in my bones.

I have long been a hanger-on at surgical meetings, hoping to meet those mighty figures that shaped surgical history. I saw W. Dean Warren once and had a very long hour with the great Mark Ravitch. Oliver Beahrs once performed magic tricks at a dinner I attended. At every surgical meeting there is an old guy (and now occasionally with the change in our profession, an elderly lady) getting on the bus to go to the reception or dinner dance. Often they are alone, their spouses having departed before them. As a young man, I wondered why the heck they came to the meetings. Just like every generation before, ours was eager to grab the reins, and in our ardor for future glory, we were polite but also restless for them to move aside. I hadn’t yet learned the importance of history and of listening.

What I missed while carousing with my young colleagues was an opportunity to hear history first hand and to learn that, what we thought was so cutting edge, these men and women had long ago considered. Many of our living legends imagined some of today’s innovations but they lacked the technology to bring their dreams to fruition, or time and age defeated them before they reached the final chapter of their research. It was when I was about 50 that I wised up and began seeking out living legends like Frank Moody and Frank Spencer.

In the case of Frank Moody, he was quite elderly when I first met him. For some reason, he knew who I was and shook my hand softly. I didn’t recognize him initially, but at the sound of his name, I knew I was in the presence of a major figure in 20th century gastrointestinal surgery. He had been at the University of California, San Francisco, during an historic time when George Sheldon, Donald Trunkey and other great surgeons trained there with J. Englebert Dunphy as their chief. Dr. Moody’s CV lists 141 articles in basic and clinical science that have had a profound impact on how we view the gastrointestinal tract. He was Chief at the University of Utah and the University of Alabama and finished his career as professor at the University of Texas-Houston. His awards and achievements were legion.

Parkinson’s had only recently really begun to affect him when I met him, and as the years went by his voice became so very faint that I had to lean in to hear him. We would sit together at the back of the dinner dance room so that we could hear each other. And while the other guests entertained themselves, Dr. Moody and I would discuss his life, scientific method and philosophy as well as his insights into his own case of Parkinsonism. I would see him at meetings, making his way slowly but steadily along a corridor while others briskly walked by, unaware that the man they just passed was among the most important surgical pioneers of our time. It was not sad that Dr. Moody was elderly and unrecognized, but that we younger surgeons missed knowing a great man in our tendency to rush past history.

 

 

History is not facts and dates, but rather, it is people and their lives. Yes, the history of our profession is embodied by pioneers like Frank Moody and the others I’ve mentioned.

We have many Fellows among us who are living history, still contributing – maybe not at the dais but at the dinner table, speaking softly and walking a bit slower than their juniors. Thanks to LaMar McGinnis who started it and Don Nakayama who continues it, the College has a History Community on the ACS Communities, an active Surgical History Group, and a will to acknowledge the history that lives and breathes among us. The Surgical History Group has organized a full program of events at the Clinical Congress and I hope many attendees take the opportunity to attend.

Take a moment at your next meeting or at the Clinical Congress and look for those historic surgeons still with us. Be smarter than I was at a young age and get to know them. You may learn something from them you can’t learn anyplace else.

Dr. Hughes is clinical professor in the department of surgery and director of medical education at the Kansas University School of Medicine, Salina Campus, and Co-Editor of ACS Surgery News.

Publications
Sections

The email came with the words, “It is with sadness we report that Frank Moody died. …” I was instantly transported to the last time I saw the man and a flood of emotions swept over me. The name Frank Moody will ring a distant bell or none at all to some in our profession. Like many of the greats of surgery, he belongs to the ages.

I remember the first time I asked a student, “Who is Michael DeBakey?” I was dumbfounded to be greeted with a blank stare. How could a student of medicine not know of Dr. DeBakey? A few years later, the same question prompted a smart aleck reply that he was the man who invented DeBakey forceps. Well, of course he did invent the forceps, but to know nothing further of the man who was the world’s expert on ulcer disease in the 1940s, the progenitor of the National Medical Library, and among the foremost pioneers of heart surgery seemed beyond belief.

 

Photo credit: Dwight Andrews/UTHealth
Dr. Frank Moody

My mentor, Ernest Poulos, has long since left the active surgical scene. At times he would note the passing of one of his heroes like Carl Moyer (look it up!) and say, “Sic transit gloria mundi.” At 27 and anxious to get the right to cut into my fellow human beings, I would cock my head like a confounded puppy and wonder what that meant. I looked up the translation and meaning long ago, but now with age I understand the phrase in my bones.

I have long been a hanger-on at surgical meetings, hoping to meet those mighty figures that shaped surgical history. I saw W. Dean Warren once and had a very long hour with the great Mark Ravitch. Oliver Beahrs once performed magic tricks at a dinner I attended. At every surgical meeting there is an old guy (and now occasionally with the change in our profession, an elderly lady) getting on the bus to go to the reception or dinner dance. Often they are alone, their spouses having departed before them. As a young man, I wondered why the heck they came to the meetings. Just like every generation before, ours was eager to grab the reins, and in our ardor for future glory, we were polite but also restless for them to move aside. I hadn’t yet learned the importance of history and of listening.

What I missed while carousing with my young colleagues was an opportunity to hear history first hand and to learn that, what we thought was so cutting edge, these men and women had long ago considered. Many of our living legends imagined some of today’s innovations but they lacked the technology to bring their dreams to fruition, or time and age defeated them before they reached the final chapter of their research. It was when I was about 50 that I wised up and began seeking out living legends like Frank Moody and Frank Spencer.

In the case of Frank Moody, he was quite elderly when I first met him. For some reason, he knew who I was and shook my hand softly. I didn’t recognize him initially, but at the sound of his name, I knew I was in the presence of a major figure in 20th century gastrointestinal surgery. He had been at the University of California, San Francisco, during an historic time when George Sheldon, Donald Trunkey and other great surgeons trained there with J. Englebert Dunphy as their chief. Dr. Moody’s CV lists 141 articles in basic and clinical science that have had a profound impact on how we view the gastrointestinal tract. He was Chief at the University of Utah and the University of Alabama and finished his career as professor at the University of Texas-Houston. His awards and achievements were legion.

Parkinson’s had only recently really begun to affect him when I met him, and as the years went by his voice became so very faint that I had to lean in to hear him. We would sit together at the back of the dinner dance room so that we could hear each other. And while the other guests entertained themselves, Dr. Moody and I would discuss his life, scientific method and philosophy as well as his insights into his own case of Parkinsonism. I would see him at meetings, making his way slowly but steadily along a corridor while others briskly walked by, unaware that the man they just passed was among the most important surgical pioneers of our time. It was not sad that Dr. Moody was elderly and unrecognized, but that we younger surgeons missed knowing a great man in our tendency to rush past history.

 

 

History is not facts and dates, but rather, it is people and their lives. Yes, the history of our profession is embodied by pioneers like Frank Moody and the others I’ve mentioned.

We have many Fellows among us who are living history, still contributing – maybe not at the dais but at the dinner table, speaking softly and walking a bit slower than their juniors. Thanks to LaMar McGinnis who started it and Don Nakayama who continues it, the College has a History Community on the ACS Communities, an active Surgical History Group, and a will to acknowledge the history that lives and breathes among us. The Surgical History Group has organized a full program of events at the Clinical Congress and I hope many attendees take the opportunity to attend.

Take a moment at your next meeting or at the Clinical Congress and look for those historic surgeons still with us. Be smarter than I was at a young age and get to know them. You may learn something from them you can’t learn anyplace else.

Dr. Hughes is clinical professor in the department of surgery and director of medical education at the Kansas University School of Medicine, Salina Campus, and Co-Editor of ACS Surgery News.

The email came with the words, “It is with sadness we report that Frank Moody died. …” I was instantly transported to the last time I saw the man and a flood of emotions swept over me. The name Frank Moody will ring a distant bell or none at all to some in our profession. Like many of the greats of surgery, he belongs to the ages.

I remember the first time I asked a student, “Who is Michael DeBakey?” I was dumbfounded to be greeted with a blank stare. How could a student of medicine not know of Dr. DeBakey? A few years later, the same question prompted a smart aleck reply that he was the man who invented DeBakey forceps. Well, of course he did invent the forceps, but to know nothing further of the man who was the world’s expert on ulcer disease in the 1940s, the progenitor of the National Medical Library, and among the foremost pioneers of heart surgery seemed beyond belief.

 

Photo credit: Dwight Andrews/UTHealth
Dr. Frank Moody

My mentor, Ernest Poulos, has long since left the active surgical scene. At times he would note the passing of one of his heroes like Carl Moyer (look it up!) and say, “Sic transit gloria mundi.” At 27 and anxious to get the right to cut into my fellow human beings, I would cock my head like a confounded puppy and wonder what that meant. I looked up the translation and meaning long ago, but now with age I understand the phrase in my bones.

I have long been a hanger-on at surgical meetings, hoping to meet those mighty figures that shaped surgical history. I saw W. Dean Warren once and had a very long hour with the great Mark Ravitch. Oliver Beahrs once performed magic tricks at a dinner I attended. At every surgical meeting there is an old guy (and now occasionally with the change in our profession, an elderly lady) getting on the bus to go to the reception or dinner dance. Often they are alone, their spouses having departed before them. As a young man, I wondered why the heck they came to the meetings. Just like every generation before, ours was eager to grab the reins, and in our ardor for future glory, we were polite but also restless for them to move aside. I hadn’t yet learned the importance of history and of listening.

What I missed while carousing with my young colleagues was an opportunity to hear history first hand and to learn that, what we thought was so cutting edge, these men and women had long ago considered. Many of our living legends imagined some of today’s innovations but they lacked the technology to bring their dreams to fruition, or time and age defeated them before they reached the final chapter of their research. It was when I was about 50 that I wised up and began seeking out living legends like Frank Moody and Frank Spencer.

In the case of Frank Moody, he was quite elderly when I first met him. For some reason, he knew who I was and shook my hand softly. I didn’t recognize him initially, but at the sound of his name, I knew I was in the presence of a major figure in 20th century gastrointestinal surgery. He had been at the University of California, San Francisco, during an historic time when George Sheldon, Donald Trunkey and other great surgeons trained there with J. Englebert Dunphy as their chief. Dr. Moody’s CV lists 141 articles in basic and clinical science that have had a profound impact on how we view the gastrointestinal tract. He was Chief at the University of Utah and the University of Alabama and finished his career as professor at the University of Texas-Houston. His awards and achievements were legion.

Parkinson’s had only recently really begun to affect him when I met him, and as the years went by his voice became so very faint that I had to lean in to hear him. We would sit together at the back of the dinner dance room so that we could hear each other. And while the other guests entertained themselves, Dr. Moody and I would discuss his life, scientific method and philosophy as well as his insights into his own case of Parkinsonism. I would see him at meetings, making his way slowly but steadily along a corridor while others briskly walked by, unaware that the man they just passed was among the most important surgical pioneers of our time. It was not sad that Dr. Moody was elderly and unrecognized, but that we younger surgeons missed knowing a great man in our tendency to rush past history.

 

 

History is not facts and dates, but rather, it is people and their lives. Yes, the history of our profession is embodied by pioneers like Frank Moody and the others I’ve mentioned.

We have many Fellows among us who are living history, still contributing – maybe not at the dais but at the dinner table, speaking softly and walking a bit slower than their juniors. Thanks to LaMar McGinnis who started it and Don Nakayama who continues it, the College has a History Community on the ACS Communities, an active Surgical History Group, and a will to acknowledge the history that lives and breathes among us. The Surgical History Group has organized a full program of events at the Clinical Congress and I hope many attendees take the opportunity to attend.

Take a moment at your next meeting or at the Clinical Congress and look for those historic surgeons still with us. Be smarter than I was at a young age and get to know them. You may learn something from them you can’t learn anyplace else.

Dr. Hughes is clinical professor in the department of surgery and director of medical education at the Kansas University School of Medicine, Salina Campus, and Co-Editor of ACS Surgery News.

Publications
Publications
Article Type
Display Headline
Sic transit gloria mundi
Display Headline
Sic transit gloria mundi
Sections
Disallow All Ads

Striking the balance: Who should be screened for CP-CRE acquisition?

Article Type
Changed
Fri, 01/18/2019 - 16:15

 

Carbapenem-resistant Enterobacteriaceae (CRE) are extremely drug-resistant organisms. According to the Centers for Disease Control and Prevention’s National Healthcare Safety Network, in 2014 in the United States, 3.6% of Enterobacteriaceae causing hospital-acquired infections were resistant to carbapenems.1 Antibiotic treatment options for CRE infections are severely limited, and mortality for invasive infections can be as high as 40%-50%.2

Resistance to carbapenems can be mediated by several mechanisms. From an epidemiologic standpoint, production of carbapenemases is the most-threatening mechanism because Enterobacteriaceae-harboring carbapenemases are highly transmissible.

Carbapenemase-producing CRE (CP-CRE) have caused large outbreaks throughout the world. Israel experienced a nationwide outbreak of CP-CRE, primarily Klebsiella pneumoniae carbapenemase–producing Klebsiella pneumoniae, in the mid-2000s. At the peak of the outbreak in 2007, there were 185 new cases per month (55.5/100,000 patient-days). A successful intervention at the national level dramatically decreased the incidence to 4.8/100,000 patient days in 2012.3

One component of the intervention (which is still ongoing) is active surveillance of high-risk groups using rectal swabs. Upon admission to the hospital, we screen patients who were recently in other hospitals or long-term care facilities. In addition, when a patient is newly diagnosed with CP-CRE (either asymptomatic carriage or clinical infection), we screen patients who had contact with that index case before isolation measures were implemented.

We recently published a study in Infection Control and Hospital Epidemiology that draws on our experience with CP-CRE screening of contacts at Tel Aviv Sourasky Medical Center.4 Both Israeli and International guidelines do not precisely define which contacts of a CP-CRE index case warrant screening. For example, should only roommates of index cases be screened or should we screen all patients on the same ward as the index case? Likewise, is there a minimum time of contact that should trigger screening?

Identifying which contacts are at high risk of acquiring CP-CRE is important for two reasons: We want to detect contacts who acquired CP-CRE so that they can be isolated before further transmission occurs, and we don’t want to waste resources and screen those at low risk. In our hospital, the criteria for being a contact are staying in the same ward and being treated by the same nursing staff as a newly identified CP-CRE patient.

This strategy appears to lead to overscreening, as we found that from October 2008 to June 2012, 3,158 screening tests were performed to detect 53 positive contacts (a yield of less than 2%). In order to screen more efficiently, our study aimed to determine risk factors for CP-CRE acquisition among patients exposed to a CP-CRE index patient.

We used a matched case-control design. The case group consisted of the 53 contacts who screened positive for CP-CRE. For each case we chose 2 controls: contacts who screened negative for CP-CRE. The basis for matching between the case and the 2 controls was that they were exposed to the same index patient. The benefit of matching this way was that it eliminated the question of whether a contact became positive because the index patient was more likely to transmit CP-CRE (e.g., because of diarrhea), and not because of characteristics of the contact patients themselves.

We found three factors that increased the risk that a contact would screen positive:

• Contact period of at least 3 days with the index case.

• Being on mechanical ventilation.

• Having a history of carriage or infection with another multidrug-resistant organism (such as methicillin-resistant Staphylococcus aureus).

Unexpectedly, sharing a room with the index patient or being debilitated did not significantly increase the risk of acquiring CP-CRE.

Many studies have identified antibiotic use as a risk factor for acquiring CP-CRE. In our study, no class of antibiotic increased the risk of CP-CRE acquisition, probably because only a small number of patients received each class. We were surprised to find that contacts who had taken cephalosporins were less likely to acquire CP-CRE. On further examination, when we compared patients who received only cephalosporins with patients who received no antibiotic, this protective effect disappeared. Nevertheless, compared with other antibiotics, it appears that cephalosporins might pose less of a risk for CP-CRE acquisition. More studies are needed to confirm our findings.

Our findings have practical implications for infection control. Using the risk factors we identified could help us to avoid excessive screening. We calculated that selective screening, based on our three risk factors, would have decreased the number of contacts screened by 30%, but 2 out of 53 positive contacts would have been missed. Institutions need to decide whether that is a trade-off they are willing to make.

Another way to apply our findings could be to add an additional layer of infection control by preemptively implementing contact precautions for patients at highest risk, for example, those with more than one risk factor.
 

 

 

1. Weiner LM, Fridkin SK, Aponte-Torres Z, Avery L, Coffin N, Dudeck MA, Edwards JR, Jernigan JA, Konnor R, Soe MM, Peterson K, Clifford McDonald L. Vital signs: preventing antibiotic-resistant infections in hospitals - United States, 2014. Am J Transplant. 2016 Jul;16(7):2224-30.

2. Centers for Disease Control and Prevention. Facility guidance for control of carbapenem-resistant Enterobacteriaceae (CRE): November 2015 update – CRE Toolkit.

3. Schwaber MJ, Carmeli Y. An ongoing national intervention to contain the spread of carbapenem-resistant enterobacteriaceae. Clin Infect Dis. 2014 Mar;58(5):697-703.

Schwartz-Neiderman A, Braun T, Fallach N, Schwartz D, Carmeli Y, Schechner V. Risk factors for carbapenemase-producing carbapenem-resistant Enterobacteriaceae (CP-CRE) acquisition among contacts of newly diagnosed CP-CRE patients. Infect Control Hosp Epidemiol. 2016 Jul 25:1-7.
 

Vered Schechner, MD, MSc, is an infection control physician in the department of epidemiology at Tel Aviv Sourasky Medical Center.

Publications
Topics
Sections

 

Carbapenem-resistant Enterobacteriaceae (CRE) are extremely drug-resistant organisms. According to the Centers for Disease Control and Prevention’s National Healthcare Safety Network, in 2014 in the United States, 3.6% of Enterobacteriaceae causing hospital-acquired infections were resistant to carbapenems.1 Antibiotic treatment options for CRE infections are severely limited, and mortality for invasive infections can be as high as 40%-50%.2

Resistance to carbapenems can be mediated by several mechanisms. From an epidemiologic standpoint, production of carbapenemases is the most-threatening mechanism because Enterobacteriaceae-harboring carbapenemases are highly transmissible.

Carbapenemase-producing CRE (CP-CRE) have caused large outbreaks throughout the world. Israel experienced a nationwide outbreak of CP-CRE, primarily Klebsiella pneumoniae carbapenemase–producing Klebsiella pneumoniae, in the mid-2000s. At the peak of the outbreak in 2007, there were 185 new cases per month (55.5/100,000 patient-days). A successful intervention at the national level dramatically decreased the incidence to 4.8/100,000 patient days in 2012.3

One component of the intervention (which is still ongoing) is active surveillance of high-risk groups using rectal swabs. Upon admission to the hospital, we screen patients who were recently in other hospitals or long-term care facilities. In addition, when a patient is newly diagnosed with CP-CRE (either asymptomatic carriage or clinical infection), we screen patients who had contact with that index case before isolation measures were implemented.

We recently published a study in Infection Control and Hospital Epidemiology that draws on our experience with CP-CRE screening of contacts at Tel Aviv Sourasky Medical Center.4 Both Israeli and International guidelines do not precisely define which contacts of a CP-CRE index case warrant screening. For example, should only roommates of index cases be screened or should we screen all patients on the same ward as the index case? Likewise, is there a minimum time of contact that should trigger screening?

Identifying which contacts are at high risk of acquiring CP-CRE is important for two reasons: We want to detect contacts who acquired CP-CRE so that they can be isolated before further transmission occurs, and we don’t want to waste resources and screen those at low risk. In our hospital, the criteria for being a contact are staying in the same ward and being treated by the same nursing staff as a newly identified CP-CRE patient.

This strategy appears to lead to overscreening, as we found that from October 2008 to June 2012, 3,158 screening tests were performed to detect 53 positive contacts (a yield of less than 2%). In order to screen more efficiently, our study aimed to determine risk factors for CP-CRE acquisition among patients exposed to a CP-CRE index patient.

We used a matched case-control design. The case group consisted of the 53 contacts who screened positive for CP-CRE. For each case we chose 2 controls: contacts who screened negative for CP-CRE. The basis for matching between the case and the 2 controls was that they were exposed to the same index patient. The benefit of matching this way was that it eliminated the question of whether a contact became positive because the index patient was more likely to transmit CP-CRE (e.g., because of diarrhea), and not because of characteristics of the contact patients themselves.

We found three factors that increased the risk that a contact would screen positive:

• Contact period of at least 3 days with the index case.

• Being on mechanical ventilation.

• Having a history of carriage or infection with another multidrug-resistant organism (such as methicillin-resistant Staphylococcus aureus).

Unexpectedly, sharing a room with the index patient or being debilitated did not significantly increase the risk of acquiring CP-CRE.

Many studies have identified antibiotic use as a risk factor for acquiring CP-CRE. In our study, no class of antibiotic increased the risk of CP-CRE acquisition, probably because only a small number of patients received each class. We were surprised to find that contacts who had taken cephalosporins were less likely to acquire CP-CRE. On further examination, when we compared patients who received only cephalosporins with patients who received no antibiotic, this protective effect disappeared. Nevertheless, compared with other antibiotics, it appears that cephalosporins might pose less of a risk for CP-CRE acquisition. More studies are needed to confirm our findings.

Our findings have practical implications for infection control. Using the risk factors we identified could help us to avoid excessive screening. We calculated that selective screening, based on our three risk factors, would have decreased the number of contacts screened by 30%, but 2 out of 53 positive contacts would have been missed. Institutions need to decide whether that is a trade-off they are willing to make.

Another way to apply our findings could be to add an additional layer of infection control by preemptively implementing contact precautions for patients at highest risk, for example, those with more than one risk factor.
 

 

 

1. Weiner LM, Fridkin SK, Aponte-Torres Z, Avery L, Coffin N, Dudeck MA, Edwards JR, Jernigan JA, Konnor R, Soe MM, Peterson K, Clifford McDonald L. Vital signs: preventing antibiotic-resistant infections in hospitals - United States, 2014. Am J Transplant. 2016 Jul;16(7):2224-30.

2. Centers for Disease Control and Prevention. Facility guidance for control of carbapenem-resistant Enterobacteriaceae (CRE): November 2015 update – CRE Toolkit.

3. Schwaber MJ, Carmeli Y. An ongoing national intervention to contain the spread of carbapenem-resistant enterobacteriaceae. Clin Infect Dis. 2014 Mar;58(5):697-703.

Schwartz-Neiderman A, Braun T, Fallach N, Schwartz D, Carmeli Y, Schechner V. Risk factors for carbapenemase-producing carbapenem-resistant Enterobacteriaceae (CP-CRE) acquisition among contacts of newly diagnosed CP-CRE patients. Infect Control Hosp Epidemiol. 2016 Jul 25:1-7.
 

Vered Schechner, MD, MSc, is an infection control physician in the department of epidemiology at Tel Aviv Sourasky Medical Center.

 

Carbapenem-resistant Enterobacteriaceae (CRE) are extremely drug-resistant organisms. According to the Centers for Disease Control and Prevention’s National Healthcare Safety Network, in 2014 in the United States, 3.6% of Enterobacteriaceae causing hospital-acquired infections were resistant to carbapenems.1 Antibiotic treatment options for CRE infections are severely limited, and mortality for invasive infections can be as high as 40%-50%.2

Resistance to carbapenems can be mediated by several mechanisms. From an epidemiologic standpoint, production of carbapenemases is the most-threatening mechanism because Enterobacteriaceae-harboring carbapenemases are highly transmissible.

Carbapenemase-producing CRE (CP-CRE) have caused large outbreaks throughout the world. Israel experienced a nationwide outbreak of CP-CRE, primarily Klebsiella pneumoniae carbapenemase–producing Klebsiella pneumoniae, in the mid-2000s. At the peak of the outbreak in 2007, there were 185 new cases per month (55.5/100,000 patient-days). A successful intervention at the national level dramatically decreased the incidence to 4.8/100,000 patient days in 2012.3

One component of the intervention (which is still ongoing) is active surveillance of high-risk groups using rectal swabs. Upon admission to the hospital, we screen patients who were recently in other hospitals or long-term care facilities. In addition, when a patient is newly diagnosed with CP-CRE (either asymptomatic carriage or clinical infection), we screen patients who had contact with that index case before isolation measures were implemented.

We recently published a study in Infection Control and Hospital Epidemiology that draws on our experience with CP-CRE screening of contacts at Tel Aviv Sourasky Medical Center.4 Both Israeli and International guidelines do not precisely define which contacts of a CP-CRE index case warrant screening. For example, should only roommates of index cases be screened or should we screen all patients on the same ward as the index case? Likewise, is there a minimum time of contact that should trigger screening?

Identifying which contacts are at high risk of acquiring CP-CRE is important for two reasons: We want to detect contacts who acquired CP-CRE so that they can be isolated before further transmission occurs, and we don’t want to waste resources and screen those at low risk. In our hospital, the criteria for being a contact are staying in the same ward and being treated by the same nursing staff as a newly identified CP-CRE patient.

This strategy appears to lead to overscreening, as we found that from October 2008 to June 2012, 3,158 screening tests were performed to detect 53 positive contacts (a yield of less than 2%). In order to screen more efficiently, our study aimed to determine risk factors for CP-CRE acquisition among patients exposed to a CP-CRE index patient.

We used a matched case-control design. The case group consisted of the 53 contacts who screened positive for CP-CRE. For each case we chose 2 controls: contacts who screened negative for CP-CRE. The basis for matching between the case and the 2 controls was that they were exposed to the same index patient. The benefit of matching this way was that it eliminated the question of whether a contact became positive because the index patient was more likely to transmit CP-CRE (e.g., because of diarrhea), and not because of characteristics of the contact patients themselves.

We found three factors that increased the risk that a contact would screen positive:

• Contact period of at least 3 days with the index case.

• Being on mechanical ventilation.

• Having a history of carriage or infection with another multidrug-resistant organism (such as methicillin-resistant Staphylococcus aureus).

Unexpectedly, sharing a room with the index patient or being debilitated did not significantly increase the risk of acquiring CP-CRE.

Many studies have identified antibiotic use as a risk factor for acquiring CP-CRE. In our study, no class of antibiotic increased the risk of CP-CRE acquisition, probably because only a small number of patients received each class. We were surprised to find that contacts who had taken cephalosporins were less likely to acquire CP-CRE. On further examination, when we compared patients who received only cephalosporins with patients who received no antibiotic, this protective effect disappeared. Nevertheless, compared with other antibiotics, it appears that cephalosporins might pose less of a risk for CP-CRE acquisition. More studies are needed to confirm our findings.

Our findings have practical implications for infection control. Using the risk factors we identified could help us to avoid excessive screening. We calculated that selective screening, based on our three risk factors, would have decreased the number of contacts screened by 30%, but 2 out of 53 positive contacts would have been missed. Institutions need to decide whether that is a trade-off they are willing to make.

Another way to apply our findings could be to add an additional layer of infection control by preemptively implementing contact precautions for patients at highest risk, for example, those with more than one risk factor.
 

 

 

1. Weiner LM, Fridkin SK, Aponte-Torres Z, Avery L, Coffin N, Dudeck MA, Edwards JR, Jernigan JA, Konnor R, Soe MM, Peterson K, Clifford McDonald L. Vital signs: preventing antibiotic-resistant infections in hospitals - United States, 2014. Am J Transplant. 2016 Jul;16(7):2224-30.

2. Centers for Disease Control and Prevention. Facility guidance for control of carbapenem-resistant Enterobacteriaceae (CRE): November 2015 update – CRE Toolkit.

3. Schwaber MJ, Carmeli Y. An ongoing national intervention to contain the spread of carbapenem-resistant enterobacteriaceae. Clin Infect Dis. 2014 Mar;58(5):697-703.

Schwartz-Neiderman A, Braun T, Fallach N, Schwartz D, Carmeli Y, Schechner V. Risk factors for carbapenemase-producing carbapenem-resistant Enterobacteriaceae (CP-CRE) acquisition among contacts of newly diagnosed CP-CRE patients. Infect Control Hosp Epidemiol. 2016 Jul 25:1-7.
 

Vered Schechner, MD, MSc, is an infection control physician in the department of epidemiology at Tel Aviv Sourasky Medical Center.

Publications
Publications
Topics
Article Type
Sections
Disallow All Ads
Alternative CME

Little drops of gold

Article Type
Changed
Fri, 01/18/2019 - 16:15

 

Is it more difficult to get blood from a stone or urine from a 3-month-old infant with a fever for which there is no apparent cause? Silly question? Not if you’re a pediatrician, and it’s 4:30 on a Friday afternoon before a 3-day holiday weekend.

You would probably prefer your chances with the stone. You have been there before. You have been peed on more than once by a 3-month-old baby you were examining. But you know from experience that when you really need just a milliliter or two of urine from a sick infant to rule out a diagnosis, those few drops of golden liquid will be hard to come by.

Dr. William G. Wilkoff practiced primary care pediatrics in Brunswick, Maine, for nearly 40 years.
Dr. William G. Wilkoff
What are your options? You can reexamine the infant and hope that her tympanic membranes that looked so normal 20 minutes ago have now become red, opaque, and bulging. Or maybe you will hear a few crackles in her chest that you didn’t hear on first listen. Any hint of a diagnosis other than a urinary tract infection could make the results of a urine sample moot.

But of course the child’s exam hasn’t changed, and you can’t convince yourself that your training can be ignored. You must have that urine. Can you bring yourself to launch an invasive attack on the child with a catheter? Despite your reassurances and explanations and your confidence with the technique, catheterization isn’t easy with the child’s parents watching. You wonder again, “Do I really need that urine?” You have done one or two needle bladder aspirations during your training years ago, but that prospect has even less appeal than the catheterization. Of course, there is always the urine bag and its significant risk of providing you with a contaminated sample or leaking even if it has been properly applied.

If only the patient were old enough to follow directions and give you a clean catch midstream sample. But you have chosen to be a pediatrician, and with that comes the reality that most of your sick young patients with unexplained fevers aren’t going to be able to comply by producing a urine sample. Sometimes you get lucky, and as the child is being prepped for catheterization or application of the collecting bag, she will surprise you by squirting out a small arc of urine that can be caught in midair – that is, if you or your assistant is prepared with an open sterile (or even just clean) cup and quick hands. After several missed opportunities over the first several years in practice, I have tried to remember to always have my assistants ready with an open container. And remind them to keep their eyes on the exposed perineum of any infant from whom we might need a clean urine sample.

But there is another option, and you can find it in this September’s Pediatrics (Evaluation of a New Strategy for Clean-Catch Urine in Infants, Labrosse et al. 2016 Sept;138[3]). The Canadian investigators describe a technique in which the infant is stimulated to void. After giving the child 20 minutes to drink and gently cleaning the perineum, the child is held vertically, the girls with their hips flexed. The physician or nurse then taps the suprapubic area at a rate of 100 taps per minute for 30 seconds and then gently massages the lumbar paravertebral area for 30 seconds. The two stimulation maneuvers are then alternated until the child voids. The investigators recommend stopping if no urine is obtained in 300 seconds, or 5 minutes.

The results are very encouraging with a success rate of 49% on a series of 126 infants. The investigators report a contamination rate of 16% that is not statistically different from collections using an invasive technique. Median time to success was 45 seconds.

You can look at the photos for yourself, but it looks like you would need at least one assistant in addition to a parent who is holding the child. I suspect that it also helps to have quick hands once the voiding starts.

It certainly sounds like a technique worth trying. The authors claim that when used as the first attempt at collection, the number of catheterizations could be cut by a third. I suspect that just like with any technique, some folks on your staff will emerge as the ones with the magic hands and might have a success rate well above that reported in this article. Groom and treasure those in-house experts at collecting those little yellow drops. They are worth their weight in gold.
 

 

Dr. Wilkoff practiced primary care pediatrics in Brunswick, Maine, for nearly 40 years. He has authored several books on behavioral pediatrics including “How to Say No to Your Toddler.” Email him at [email protected].

Publications
Topics
Sections

 

Is it more difficult to get blood from a stone or urine from a 3-month-old infant with a fever for which there is no apparent cause? Silly question? Not if you’re a pediatrician, and it’s 4:30 on a Friday afternoon before a 3-day holiday weekend.

You would probably prefer your chances with the stone. You have been there before. You have been peed on more than once by a 3-month-old baby you were examining. But you know from experience that when you really need just a milliliter or two of urine from a sick infant to rule out a diagnosis, those few drops of golden liquid will be hard to come by.

Dr. William G. Wilkoff practiced primary care pediatrics in Brunswick, Maine, for nearly 40 years.
Dr. William G. Wilkoff
What are your options? You can reexamine the infant and hope that her tympanic membranes that looked so normal 20 minutes ago have now become red, opaque, and bulging. Or maybe you will hear a few crackles in her chest that you didn’t hear on first listen. Any hint of a diagnosis other than a urinary tract infection could make the results of a urine sample moot.

But of course the child’s exam hasn’t changed, and you can’t convince yourself that your training can be ignored. You must have that urine. Can you bring yourself to launch an invasive attack on the child with a catheter? Despite your reassurances and explanations and your confidence with the technique, catheterization isn’t easy with the child’s parents watching. You wonder again, “Do I really need that urine?” You have done one or two needle bladder aspirations during your training years ago, but that prospect has even less appeal than the catheterization. Of course, there is always the urine bag and its significant risk of providing you with a contaminated sample or leaking even if it has been properly applied.

If only the patient were old enough to follow directions and give you a clean catch midstream sample. But you have chosen to be a pediatrician, and with that comes the reality that most of your sick young patients with unexplained fevers aren’t going to be able to comply by producing a urine sample. Sometimes you get lucky, and as the child is being prepped for catheterization or application of the collecting bag, she will surprise you by squirting out a small arc of urine that can be caught in midair – that is, if you or your assistant is prepared with an open sterile (or even just clean) cup and quick hands. After several missed opportunities over the first several years in practice, I have tried to remember to always have my assistants ready with an open container. And remind them to keep their eyes on the exposed perineum of any infant from whom we might need a clean urine sample.

But there is another option, and you can find it in this September’s Pediatrics (Evaluation of a New Strategy for Clean-Catch Urine in Infants, Labrosse et al. 2016 Sept;138[3]). The Canadian investigators describe a technique in which the infant is stimulated to void. After giving the child 20 minutes to drink and gently cleaning the perineum, the child is held vertically, the girls with their hips flexed. The physician or nurse then taps the suprapubic area at a rate of 100 taps per minute for 30 seconds and then gently massages the lumbar paravertebral area for 30 seconds. The two stimulation maneuvers are then alternated until the child voids. The investigators recommend stopping if no urine is obtained in 300 seconds, or 5 minutes.

The results are very encouraging with a success rate of 49% on a series of 126 infants. The investigators report a contamination rate of 16% that is not statistically different from collections using an invasive technique. Median time to success was 45 seconds.

You can look at the photos for yourself, but it looks like you would need at least one assistant in addition to a parent who is holding the child. I suspect that it also helps to have quick hands once the voiding starts.

It certainly sounds like a technique worth trying. The authors claim that when used as the first attempt at collection, the number of catheterizations could be cut by a third. I suspect that just like with any technique, some folks on your staff will emerge as the ones with the magic hands and might have a success rate well above that reported in this article. Groom and treasure those in-house experts at collecting those little yellow drops. They are worth their weight in gold.
 

 

Dr. Wilkoff practiced primary care pediatrics in Brunswick, Maine, for nearly 40 years. He has authored several books on behavioral pediatrics including “How to Say No to Your Toddler.” Email him at [email protected].

 

Is it more difficult to get blood from a stone or urine from a 3-month-old infant with a fever for which there is no apparent cause? Silly question? Not if you’re a pediatrician, and it’s 4:30 on a Friday afternoon before a 3-day holiday weekend.

You would probably prefer your chances with the stone. You have been there before. You have been peed on more than once by a 3-month-old baby you were examining. But you know from experience that when you really need just a milliliter or two of urine from a sick infant to rule out a diagnosis, those few drops of golden liquid will be hard to come by.

Dr. William G. Wilkoff practiced primary care pediatrics in Brunswick, Maine, for nearly 40 years.
Dr. William G. Wilkoff
What are your options? You can reexamine the infant and hope that her tympanic membranes that looked so normal 20 minutes ago have now become red, opaque, and bulging. Or maybe you will hear a few crackles in her chest that you didn’t hear on first listen. Any hint of a diagnosis other than a urinary tract infection could make the results of a urine sample moot.

But of course the child’s exam hasn’t changed, and you can’t convince yourself that your training can be ignored. You must have that urine. Can you bring yourself to launch an invasive attack on the child with a catheter? Despite your reassurances and explanations and your confidence with the technique, catheterization isn’t easy with the child’s parents watching. You wonder again, “Do I really need that urine?” You have done one or two needle bladder aspirations during your training years ago, but that prospect has even less appeal than the catheterization. Of course, there is always the urine bag and its significant risk of providing you with a contaminated sample or leaking even if it has been properly applied.

If only the patient were old enough to follow directions and give you a clean catch midstream sample. But you have chosen to be a pediatrician, and with that comes the reality that most of your sick young patients with unexplained fevers aren’t going to be able to comply by producing a urine sample. Sometimes you get lucky, and as the child is being prepped for catheterization or application of the collecting bag, she will surprise you by squirting out a small arc of urine that can be caught in midair – that is, if you or your assistant is prepared with an open sterile (or even just clean) cup and quick hands. After several missed opportunities over the first several years in practice, I have tried to remember to always have my assistants ready with an open container. And remind them to keep their eyes on the exposed perineum of any infant from whom we might need a clean urine sample.

But there is another option, and you can find it in this September’s Pediatrics (Evaluation of a New Strategy for Clean-Catch Urine in Infants, Labrosse et al. 2016 Sept;138[3]). The Canadian investigators describe a technique in which the infant is stimulated to void. After giving the child 20 minutes to drink and gently cleaning the perineum, the child is held vertically, the girls with their hips flexed. The physician or nurse then taps the suprapubic area at a rate of 100 taps per minute for 30 seconds and then gently massages the lumbar paravertebral area for 30 seconds. The two stimulation maneuvers are then alternated until the child voids. The investigators recommend stopping if no urine is obtained in 300 seconds, or 5 minutes.

The results are very encouraging with a success rate of 49% on a series of 126 infants. The investigators report a contamination rate of 16% that is not statistically different from collections using an invasive technique. Median time to success was 45 seconds.

You can look at the photos for yourself, but it looks like you would need at least one assistant in addition to a parent who is holding the child. I suspect that it also helps to have quick hands once the voiding starts.

It certainly sounds like a technique worth trying. The authors claim that when used as the first attempt at collection, the number of catheterizations could be cut by a third. I suspect that just like with any technique, some folks on your staff will emerge as the ones with the magic hands and might have a success rate well above that reported in this article. Groom and treasure those in-house experts at collecting those little yellow drops. They are worth their weight in gold.
 

 

Dr. Wilkoff practiced primary care pediatrics in Brunswick, Maine, for nearly 40 years. He has authored several books on behavioral pediatrics including “How to Say No to Your Toddler.” Email him at [email protected].

Publications
Publications
Topics
Article Type
Sections
Disallow All Ads